Download as pdf or txt
Download as pdf or txt
You are on page 1of 96

Total Marks : 200

Test-24 (Subject)
( INSTA Prelims Test Series 2021 )

1. Consider the following statements regarding secularism in India


1. The Western concept of secularism embodies the positive concept of secularism.
2. The Western concept of secularism connotes a complete separation between the religion and
the state.

Which of the statements given above is/are correct?


A. 1 only
B. 2 only
C. Both 1 and 2
D. Neither 1 nor 2

Correct Answer : B

Answer Justification :

The Western concept of secularism connotes a complete separation between the


religion (the church) and the state (the politics). This negative concept of secularism is
inapplicable in the Indian situation where the society is multireligious. Hence, the Indian
Constitution embodies the positive concept of secularism, i.e., giving equal respect to
all religions or protecting all religions equally. Hence, statement 1 is incorrect.

Moreover, the Constitution has also abolished the old system of communal representation, that
is, reservation of seats in the legislatures on the basis of religion. However, it provides for the
temporary reservation of seats for the scheduled castes and scheduled tribes to ensure
adequate representation to them.

2. Which of the following is/are meaning of the word republic in Preamble?


1. Vesting of political sovereignty in the people.
2. The absence of any privileged class.

Which of the statements given above is/are correct?


A. 1 only
B. 2 only
C. Both 1 and 2
D. Neither 1 nor 2

Correct Answer : C

Answer Justification :

All the above statements are correct.

A democratic polity can be classified into two categories–monarchy and republic. In a

www.insightsactivelearn.com 1
Total Marks : 200
Test-24 (Subject)
( INSTA Prelims Test Series 2021 )

monarchy, the head of the state (usually king or queen) enjoys a hereditary position, that is, he
comes into office through succession, e.g., Britain. In a republic, on the other hand, the head
of the state is always elected directly or indirectly for a fixed period, e.g., USA.

Therefore, the term ‘republic’ in our Preamble indicates that India has an elected head called
the president. He is elected indirectly for a fixed period of five years.

A republic also means two more things: one, vesting of political sovereignty in the
people and not in a single individual like a king; second, the absence of any privileged
class and hence all public offices being opened to every citizen without any
discrimination.

3. Consider the following statements regarding Indian federation


1. Article 1 describes Indian Constitution as federal in structure.
2. The Indian federation is a union because it is indestructible.

Which of the statements given above is/are correct?


A. 1 only
B. 2 only
C. Both 1 and 2
D. Neither 1 nor 2

Correct Answer : C

Answer Justification :

All the above statements are correct.

Article 1 describes India, that is, Bharat as a ‘Union of States’ rather than a
‘Federation of States’. This provision deals with two things: one, name of the country;
and two, type of polity. There was no unanimity in the Constituent Assembly with regard to
the name of the country. Some members suggested the traditional name (Bharat), while
other advocated the modern name (India). Hence, the Constituent Assembly had to
adopt a mix of both (‘India, that is, Bharat’) Secondly, the country is described as
‘Union’ although its Constitution is federal in structure. According to Dr. B.R.
Ambedkar, the phrase ‘Union of States’ has been preferred to ‘Federation of States’ for two
reasons: one, the Indian Federation is not the result of an agreement among the states like the
American Federation; and two, the states have no right to secede from the federation. The
federation is a Union because it is indestructible. The country is an integral whole and
divided into different states only for the convenience of administration.

The term ‘federation’ is drived from a Latin word foedus which means ‘treaty’ or ‘agreement’.
Thus, a federation is a new state (political system) which is formed through a treaty or an
agreement between the various units.

www.insightsactivelearn.com 2
Total Marks : 200
Test-24 (Subject)
( INSTA Prelims Test Series 2021 )

4. A Mandamus can be issued against which of the following?

A. A tribunal
B. To enforce departmental instruction that does not possess statutory force.
C. When the duty is discretionary and not mandatory
D. To enforce a contractual obligation

Correct Answer : A

Answer Justification :

Mandamus

It literally means ‘we command’. It is a command issued by the court to a public official asking
him to perform his official duties that he has failed or refused to perform. It can also be
issued against any public body, a corporation, an inferior court, a tribunal or
government for the same purpose.

The writ of mandamus cannot be issued (a) against a private individual or body; (b) to
enforce departmental instruction that does not possess statutory force; (c) when the
duty is discretionary and not mandatory; (d) to enforce a contractual obligation; (e)
against the president of India or the state governors; and (f) against the chief justice
of a high court acting in judicial capacity.

Hence, option (a) is correct.

5. Consider the following statements regarding Right to constitutional remedies


1. The right to get the Fundamental Rights protected is in itself a fundamental right.
2. The Supreme Court, under Article 32, cannot determine a question that does not involve
Fundamental Rights.

Which of the statements given above is/are correct?


A. 1 only
B. 2 only
C. Both 1 and 2
D. Neither 1 nor 2

Correct Answer : C

Answer Justification :

All the above statements are correct.

www.insightsactivelearn.com 3
Total Marks : 200
Test-24 (Subject)
( INSTA Prelims Test Series 2021 )

Right to constitutional remedies

A mere declaration of fundamental rights in the Constitution is meaningless, useless and


worthless without providing an effective machinery for their enforcement, if and when they are
violated. Hence, Article 32 confers the right to remedies for the enforcement of the
fundamental rights of an aggrieved citizen. In other words, the right to get the
Fundamental Rights protected is in itself a fundamental right. This makes the
fundamental rights real. That is why Dr. Ambedkar called Article 32 as the most important
article of the Constitution–‘an Article without which this constitution would be a nullity. It is
the very soul of the Constitution and the very heart of it’.

It is thus clear that the Supreme Court has been constituted as the defender and guarantor of
the fundamental rights of the citizens. It has been vested with the ‘original’ and ‘wide’ powers
for that purpose.

Original, because an aggrieved citizen can directly go to the Supreme Court, not necessarily
by way of appeal. Wide, because its power is not restricted to issuing of orders or directions
but also writs of all kinds.

The purpose of Article 32 is to provide a guaranteed, effective, expeditious, inexpensive and


summary remedy for the protection of the fundamental rights. Only the Fundamental Rights
guaranteed by the Constitution can be enforced under Article 32 and not any other right like
non-fundamental constitutional rights, statutory rights, customary rights and so on. The
violation of a fundamental right is the sine qua non for the exercise of the right
conferred by Article 32. In other words, the Supreme Court, under Article 32, cannot
determine a question that does not involve Fundamental Rights. Article 32 cannot be
invoked simply to determine the constitutionality of an executive order or a legislation unless
it directly infringes any of the fundamental rights.

6. Consider the following statements regarding Article 25 of the Constitution


1. The Article 25 covers only religious beliefs (doctrines) and not the religious practices (rituals).
2. Right to propagate does not include a right to convert another person to one’s own religion.

Which of the statements given above is/are correct?


A. 1 only
B. 2 only
C. Both 1 and 2
D. Neither 1 nor 2

Correct Answer : B

Answer Justification :

Freedom of Conscience and Free Profession, Practice and Propagation of Religion

www.insightsactivelearn.com 4
Total Marks : 200
Test-24 (Subject)
( INSTA Prelims Test Series 2021 )

Article 25 says that all persons are equally entitled to freedom of conscience and the right to
freely profess, practice and propagate religion. The implications of these are:

(a) Freedom of conscience: Inner freedom of an individual to mould his relation with God or
Creatures in whatever way he desires.

(b) Right to profess: Declaration of one’s religious beliefs and faith openly and freely.

(c) Right to practice: Performance of religious worship, rituals, ceremonies and exhibition of
beliefs and ideas.

(d) Right to propagate: Transmission and dissemination of one’s religious beliefs to


others or exposition of the tenets of one’s religion. But it does not include a right to
convert another person to one’s own religion. Forcible conversions impinge on the
‘freedom of conscience’ guaranteed to all the persons alike.

From the above, it is clear that Article 25 covers not only religious beliefs (doctrines)
but also religious practices (rituals). Hence, statement 1 is incorrect.

Moreover, these rights are available to all persons–citizens as well as noncitizens. However,
these rights are subject to public order, morality, health and other provisions relating to
fundamental rights.

7. Consider the following statements regarding Article 20 of the Constitution


1. The ex-post-facto provision of Article 20 is applicable to both criminal laws and tax laws.
2. The protection against self-incrimination extends to oral evidence only and does not extend to
documentary evidence.
3. The protection against double jeopardy is available only in proceedings before a court of law
or a judicial tribunal.

Which of the statements given above is/are correct?


A. 1 only
B. 2 and 3 only
C. 3 only
D. 1, 2 and 3

Correct Answer : C

Answer Justification :

Protection in Respect of Conviction for Offences

Article 20 grants protection against arbitrary and excessive punishment to an accused person,
whether citizen or foreigner or legal person like a company or a corporation.

An ex-post-facto law is one that imposes penalties retrospectively (retroactively), that

www.insightsactivelearn.com 5
Total Marks : 200
Test-24 (Subject)
( INSTA Prelims Test Series 2021 )

is, upon acts already done or which increases the penalties for such acts. The
enactment of such a law is prohibited by the first provision of Article 20. However,
this limitation is imposed only on criminal laws and not on civil laws or tax laws.
Hence, statement 1 is incorrect.

In other words, a civil liability or a tax can be imposed retrospectively. Further, this provision
prohibits only conviction or sentence under an ex-post-facto criminal law and not the trial
thereof. Finally, the protection (immunity) under this provision cannot be claimed in case of
preventive detention or demanding security from a person.

The protection against double jeopardy is available only in proceedings before a court
of law or a judicial tribunal. In other words, it is not available in proceedings before
departmental or administrative authorities as they are not of judicial nature.

The protection against self-incrimination extends to both oral evidence and


documentary evidence. Hence, statement 2 is incorrect.

However, it does not extend to

(i) compulsory production of material objects, (ii) compulsion to give thumb impression,
specimen signature, blood specimens, and (iii) compulsory exhibition of the body. Further, it
extends only to criminal proceedings and not to civil proceedings or proceedings which are not
of criminal nature.

8. Consider the following statements regarding protection of law from Fundamental rights
1. Article 31A saves certain categories of laws from being challenged and invalidated on the
ground of contravention with any of the fundamental rights.
2. Article 31B immunizes any law included in the Ninth Schedule, on the ground of contravention
of the fundamental rights conferred by Article 14 and Article 19 only.

Which of the statements given above is/are correct?


A. 1 only
B. 2 only
C. Both 1 and 2
D. Neither 1 nor 2

Correct Answer : D

Answer Justification :

Saving of Laws Providing for Acquisition of Estates, etc.

Article 31A saves five categories of laws from being challenged and invalidated on the
ground of contravention of the fundamental rights conferred by Article 14 (equality
before law and equal protection of laws) and Article 19 (protection of six rights in

www.insightsactivelearn.com 6
Total Marks : 200
Test-24 (Subject)
( INSTA Prelims Test Series 2021 )

respect of speech, assembly, movement, etc.).

Validation of Certain Acts and Regulations

Article 31B saves the acts and regulations included in the Ninth Schedule from being
challenged and invalidated on the ground of contravention of any of the fundamental
rights. Thus, the scope of Article 31B is wider than Article 31A. Article 31B
immunizes any law included in the Ninth Schedule from all the fundamental rights
whether or not the law falls under any of the five categories specified in Article 31A.

Hence, both statement 1 and 2 are incorrect.

However, in a significant judgement delivered in I.R. Coelho case18a (2007), the Supreme
Court ruled that there could not be any blanket immunity from judicial review of laws included
in the Ninth Schedule. The court held that judicial review is a ‘basic feature’ of the
constitution and it could not be taken away by putting a law under the Ninth Schedule.

9. Which of the following are significance of fundamental rights?


1. They serve as a formidable bulwark of individual liberty.
2. They provide necessary conditions for the material and moral protection of man.
3. They protect the interests of minorities and weaker sections of society.

Which of the statements given above is/are correct?


A. 1 only
B. 2 and 3 only
C. 3 only
D. 1, 2 and 3

Correct Answer : D

Answer Justification :

All the above statements are correct.

Significance of fundamental rights

In spite of the above criticism and shortcomings, the Fundamental Rights are significant in the
following respects:

1. They constitute the bedrock of democratic system in the country.

2. They provide necessary conditions for the material and moral protection of man.

3. They serve as a formidable bulwark of individual liberty.

4. They facilitate the establishment of rule of law in the country.

www.insightsactivelearn.com 7
Total Marks : 200
Test-24 (Subject)
( INSTA Prelims Test Series 2021 )

5. They protect the interests of minorities and weaker sections of society.

6. They strengthen the secular fabric of the Indian State.

7. They check the absoluteness of the authority of the government.

8. They lay down the foundation stone of social equality and social justice.

9. They ensure the dignity and respect of individuals.

10. They facilitate the participation of people in the political and administrative process.

10. Emergency provisions in Indian constitution were borrowed from?

A. Government of India Act of 1935


B. British Constitution
C. US Constitution
D. Weimar Constitution of Germany

Correct Answer : A

Answer Justification :
Sources Features Borrowed
Federal Scheme, Office of governor, Judiciary, Public Service
Government of India
Commissions, Emergency provisions and administrative
Act of 1935
details.
Parliamentary government, Rule of Law, legislative procedure,
British Constitution single citizenship, cabinet system, prerogative writs,
parliamentary privileges and bicameralism.
Fundamental rights, independence of judiciary, judicial review,
US Constitution impeachment of the president, removal of Supreme Court and
high court judges and post of vice-president.
Weimar Constitution
Suspension of Fundamental Rights during Emergency.
of Germany

Hence, option (a) is correct.

11. The act abolished the East India Company, and transferred the powers of Government, territories and revenues
to the British Crown. Under which of the following act was this provision included?

A. Charter Act of 1813


B. Charter Act of 1833
C. Government of India Act of 1858

www.insightsactivelearn.com 8
Total Marks : 200
Test-24 (Subject)
( INSTA Prelims Test Series 2021 )

D. Indian Independence Act of 1947

Correct Answer : C

Answer Justification :

Government of India Act of 1858

This significant Act was enacted in the wake of the Revolt of 1857–also known as the
First War of Independence or the ‘sepoy mutiny’. The act known as the Act for the
Good Government of India, abolished the East India Company, and transferred the
powers of Government, territories and revenues to the British Crown. Hence, option
(c) is correct.

The features of this Act were as follows:

1. It provided that India, henceforth, was to be governed by, and in the name of, Her Majesty.
It changed the designation of the Governor-General of India to that of Viceroy of India. He
(Viceroy) was the direct representative of the British Crown in India. Lord Canning, thus,
became the first Viceroy of India.

2. It ended the system of double Government by abolishing the Board of Control and Court of
Directors.

3. It created a new office, Secretary of State for India, vested with complete authority and
control over Indian administration. The secretary of state was a member of the British Cabinet
and was responsible ultimately to the British Parliament.

4. It established a 15-member council of India to assist the Secretary of State for India. The
council was an advisory body. The secretary of state was made the Chairman of the council.

5. It constituted the Secretary of State-in Council as a body corporate, capable of suing and
being sued in India and in England.

12. Which of the following statements describe the importance of Regulating Act of 1773?
1. It recognized, for the first time, the political and administrative functions of the East India
Company
2. It laid the foundations of central administration in India.
3. The Company’s territories in India were for the first time called the ‘British possessions in
India’.

Which of the statements given above are correct?


A. 1 and 2 only
B. 2 and 3 only

www.insightsactivelearn.com 9
Total Marks : 200
Test-24 (Subject)
( INSTA Prelims Test Series 2021 )

C. 1 and 3 only
D. 1, 2 and 3

Correct Answer : A

Answer Justification :

Regulating Act of 1773

This act was of great constitutional importance as

(a) it was the first step taken by the British Government to control and regulate the
affairs of the East India Company in India;

(b) it recognized, for the first time, the political and administrative functions of the
Company; and

(c) it laid the foundations of central administration in India.

Pitt’s India Act of 1784

the act was significant for two reasons: first, the Company’s territories in India were
for the first time called the ‘British possessions in India’; and second, the British
Government was given the supreme control over Company’s affairs and its
administration in India. Hence, statement 3 is incorrect.

13. Under Fazl Ali Commission, which of the following factors were taken into account for reorganization of
states?
1. Preservation and strengthening of the unity and security of the country.
2. Linguistic and cultural homogeneity.
3. Financial, economic and administrative considerations.
4. Planning and promotion of the welfare of the people in each state as well as of the nation as a
whole.

Select the correct answer using the code given below:


A. 1, 2 and 3 only
B. 2, 3 and 4 only
C. 1, 2 and 4 only
D. 1, 2, 3 and 4

Correct Answer : D

Answer Justification :

All the above statements are correct.


www.insightsactivelearn.com 10
Total Marks : 200
Test-24 (Subject)
( INSTA Prelims Test Series 2021 )

Fazl Ali Commission

The creation of Andhra state intensified the demand from other regions for creation of states
on linguistic basis. This forced the Government of India to appoint (in December, 1953) a
threemember States Reorganisation Commission under the chairmanship of Fazl Ali to re-
examine the whole question. Its other two members were K.M. Panikkar and H.N. Kunzru. It
submitted its report in September 1955 and broadly accepted language as the basis of
reorganisation of states. But, it rejected the theory of ‘one language-one state’. Its view was
that the unity of India should be regarded as the primary consideration in any redrawing of the
country’s political units. It identified four major factors that can be taken into account
in any scheme of reorganisation of states:

(a) Preservation and strengthening of the unity and security of the country.

(b) Linguistic and cultural homogeneity.

(c) Financial, economic and administrative considerations.

(d) Planning and promotion of the welfare of the people in each state as well as of the
nation as a whole.

14. Which of the following are features of Directive Principles?


1. They aim at establishing political democracy in the country.
2. These are positive as they require the State to do certain things.
3. They promote the welfare of the community.
4. These have legal sanctions.

Select the correct answer using the code given below:


A. 1 and 4 only
B. 2 and 3 only
C. 2, 3 and 4 only
D. 1, 2, 3 and 4

Correct Answer : B

Answer Justification :

Fundamental Rights

1. These are negative as they prohibit the State from doing certain things.

2. These are justiciable, that is, they are legally enforceable by the courts in case of their
violation.

3. They aim at establishing political democracy in the country. Hence, statement 1 is


incorrect.

www.insightsactivelearn.com 11
Total Marks : 200
Test-24 (Subject)
( INSTA Prelims Test Series 2021 )

4. These have legal sanctions. Hence, statement 4 is incorrect.

5. They promote the welfare of the individual. Hence, they are personal and individualistic.

Directive Principles

1. These are positive as they require the State to do certain things.

2. These are non-justiciable, that is, they are not legally enforceable by the courts for their
violation.

3. They aim at establishing social and economic democracy in the country.

4. These have moral and political sanctions.

5. They promote the welfare of the community. Hence, they are societarian and
socialistic.

15. Which of the following are the Directives which are not included in Part IV?
1. The claims of the members of the Scheduled Castes and the Scheduled Tribes shall be taken
into consideration in the making of appointments to services and posts in connection with the
affairs of the Union or a State.
2. It shall be the endeavor of every state and every local authority within the state to provide
adequate facilities for instruction in the mother tongue.
3. It shall be the duty of the Union to promote the spread of the Hindi language and to develop it
so that it may serve as a medium of expression for all the elements of the composite culture of
India.

Which of the statements given above are correct?


A. 1 and 2 only
B. 2 and 3 only
C. 1 and 3 only
D. 1, 2 and 3

Correct Answer : D

Answer Justification :

All the above statements are correct.

Apart from the Directives included in Part IV, there are some other Directives contained in
other Parts of the Constitution. They are:

1. Claims of SCs and STs to Services: The claims of the members of the Scheduled Castes
and the Scheduled Tribes shall be taken into consideration, consistently with the maintenance
of efficiency of administration, in the making of appointments to services and posts in

www.insightsactivelearn.com 12
Total Marks : 200
Test-24 (Subject)
( INSTA Prelims Test Series 2021 )

connection with the affairs of the Union or a State (Article 335 in Part XVI).

2. Instruction in mother tongue: It shall be the endeavour of every state and every local
authority within the state to provide adequate facilities for instruction in the mother tongue at
the primary stage of education to children belonging to linguistic minority groups (Article 350-
A in Part XVII).

3. Development of the Hindi Language: It shall be the duty of the Union to promote the
spread of the Hindi language and to develop it so that it may serve as a medium of expression
for all the elements of the composite culture of India (Article 351 in Part XVII).

16. Which of the following provisions in the Constitution can be amended outside the scope of Article 368?
1. Admission or establishment of new states.
2. Abolition or creation of legislative councils in states.
3. Conferment of more jurisdiction on the Supreme Court.
4. Elections to Parliament and state legislatures.

Select the correct answer using the code given below:


A. 1 and 4 only
B. 2 and 3 only
C. 2, 3 and 4 only
D. 1, 2, 3 and 4

Correct Answer : D

Answer Justification :

All the above statements are correct.

By Simple Majority of Parliament

A number of provisions in the Constitution can be amended by a simple majority of the two
Houses of Parliament outside the scope of Article 368. These provisions include:

1. Admission or establishment of new states.

2. Formation of new states and alteration of areas, boundaries or names of existing states.

3. Abolition or creation of legislative councils in states.

4. Second Schedule–emoluments, allowances, privileges and so on of the president, the


governors, the Speakers, judges, etc.

5. Quorum in Parliament.

6. Salaries and allowances of the members of Parliament.

www.insightsactivelearn.com 13
Total Marks : 200
Test-24 (Subject)
( INSTA Prelims Test Series 2021 )

7. Rules of procedure in Parliament.

8. Privileges of the Parliament, its members and its committees.

9. Use of English language in Parliament.

10. Number of puisne judges in the Supreme Court.

11. Conferment of more jurisdiction on the Supreme Court.

12. Use of official language.

13. Elections to Parliament and state legislatures.

17. Which of the following is not a feature of presidential government of America?

A. The President is elected by Congress for a fixed tenure of four years.


B. The President governs with the help of a cabinet or a smaller body called ‘Kitchen
Cabinet’.
C. The President and his secretaries are not responsible to the Congress for their acts.
D. The American President is both the head of the State and the head of government.

Correct Answer : A

Answer Justification :

Features of presidential government

Unlike the Indian Constitution, the American Constitution provides for the presidential form of
government. The features of the American presidential system of government are as follows:

(a) The American President is both the head of the State and the head of government.
As the head of State, he occupies a ceremonial position. As the head of government, he leads
the executive organ of government.

(b) The President is elected by an electoral college for a fixed tenure of four years.
Hence, option (a) is correct.

He cannot be removed by the Congress except by impeachment for a grave unconstitutional


act.

(c) The President governs with the help of a cabinet or a smaller body called ‘Kitchen
Cabinet’. It is only an advisory body and consists of non-elected departmental secretaries.
They are selected and appointed by him, are responsible only to him, and can be removed by
him any time.

www.insightsactivelearn.com 14
Total Marks : 200
Test-24 (Subject)
( INSTA Prelims Test Series 2021 )

(d) The President and his secretaries are not responsible to the Congress for their
acts. They neither possess membership in the Congress nor attend its sessions.

(e) The President cannot dissolve the House of Representatives–the lower house of the
Congress.

(f) The doctrine of separation of powers is the basis of the American presidential system. The
legislative, executive and judicial powers of the government are separated and vested in the
three independent organs of the government.

18. Under the 42nd Amendment Act of 1976, which of the following subjects were transferred to Concurrent List
from State List?
1. Education
2. Forests
3. Weights and measures
4. Administration of justice; constitution and organization of lower courts

Select the correct answer using the code given below:


A. 1 and 4 only
B. 2 and 3 only
C. 2, 3 and 4 only
D. 1, 2, 3 and 4

Correct Answer : D

Answer Justification :

All the above statements are correct.

Both, the Parliament and state legislature can make laws with respect to any of the matters
enumerated in the Concurrent List. This list has at present 52 subjects (originally 47 subjects)
like criminal law and procedure, civil procedure, marriage and divorce, population control and
family planning, electricity, labour welfare, economic and social planning, drugs, newspapers,
books and printing press, and others. The 42nd Amendment Act of 1976 transferred five
subjects to Concurrent List from State List, that is, (a) education, (b) forests, (c) weights
and measures, (d) protection of wild animals and birds, and (e) administration of justice;
constitution and organisation of all courts except the Supreme Court and the high courts.

In case of a conflict between the Central law and the state law on a subject enumerated in the
Concurrent List, the Central law prevails over the state law. But, there is an exception. If the
state law has been reserved for the consideration of the president and has received his assent,
then the state law prevails in that state. But, it would still be competent for the Parliament to
override such a law by subsequently making a law on the same matter.

www.insightsactivelearn.com 15
Total Marks : 200
Test-24 (Subject)
( INSTA Prelims Test Series 2021 )

19. Consider the following statements regarding taxing powers of the states
1. A state legislature can impose taxes on professions, trades, callings and employments.
2. A state legislature can impose a tax on the supply of goods or services or both in case where
such supply takes place outside the state.
3. A state legislature can impose tax on the consumption or sale of electricity.

Which of the statements given above are correct?


A. 1 and 2 only
B. 2 and 3 only
C. 1 and 3 only
D. 1, 2 and 3

Correct Answer : C

Answer Justification :

The Constitution has placed the following restrictions on the taxing powers of the states:

(i) A state legislature can impose taxes on professions, trades, callings and
employments. But, the total amount of such taxes payable by any person should not exceed
₹2,500 per annum.

(ii) A state legislature is prohibited from imposing a tax on the supply of goods or
services or both in the following two cases: (a) where such supply takes place outside
the state; Hence, statement 2 is incorrect.

and (b) where such supply takes place in the course of import or export. Further, the
Parliament is empowered to formulate the principles for determining when a supply of goods
or services or both takes place outside the state, or in the course of import or export.

(iii) A state legislature can impose tax on the consumption or sale of electricity. But, no
tax can be imposed on the consumption or sale of electricity which is (a) consumed by the
Centre or sold to the Centre; or (b) consumed in the construction, maintenance or operation of
any railway by the Centre or by the concerned railway company or sold to the Centre or the
railway company for the same purpose.

(iv) A state legislature can impose a tax in respect of any water or electricity stored,
generated, consumed, distributed or sold by any authority established by Parliament for
regulating or developing any inter-state river or river valley. But, such a law, to be effective,
should be reserved for the president’s consideration and receive his assent.

20. To protect the interest of states under the Constitution, which of the following bills can be introduced in the
Parliament only on the recommendation of the President?
1. A bill which imposes or varies any tax or duty in which states are interested.
2. A bill which affects the principles on which moneys are or may be distributable to states.
www.insightsactivelearn.com 16
Total Marks : 200
Test-24 (Subject)
( INSTA Prelims Test Series 2021 )

3. A bill which imposes any surcharge on any specified tax or duty for the purpose of the Centre.

Which of the statements given above are correct?


A. 1 and 2 only
B. 2 and 3 only
C. 1 and 3 only
D. 1, 2 and 3

Correct Answer : D

Answer Justification :

Protection of the States’ Interest

To protect the interest of states in the financial matters, the Constitution lays down
that the following bills can be introduced in the Parliament only on the
recommendation of the President:

• A bill which imposes or varies any tax or duty in which states are interested;

• A bill which varies the meaning of the expression ‘agricultural income’ as defined
for the purposes of the enactments relating to Indian income tax;

• A bill which affects the principles on which moneys are or may be distributable to
states; and

• A bill which imposes any surcharge on any specified tax or duty for the purpose of
the Centre.

The expression “tax or duty in which states are interested” means: (a) a tax or duty the whole
or part of the net proceeds whereof are assigned to any state; or (b) a tax or duty by reference
to the net proceeds whereof sums are for the time being payable, out of the Consolidated Fund
of India to any state.

All the above statements are correct.

21. Who among the following is the chairman of Standing Committee of the Inter-State Council?

A. Prime minister
B. Chief minister of a state in rotation
C. Union Home Minister
D. Cabinet Secretary

www.insightsactivelearn.com 17
Total Marks : 200
Test-24 (Subject)
( INSTA Prelims Test Series 2021 )

Correct Answer : C

Answer Justification :

In pursuance of the above recommendations of the Sarkaria Commission, the Janata Dal
Government headed by V. P. Singh established the Inter-State Council in 1990. It
consists of the following members:

(i) Prime minister as the Chairman

(ii) Chief ministers of all the states

(iii) Chief ministers of union territories having legislative assemblies

(iv) Administrators of union territories not having legislative assemblies

(v) Governors of States under President’s rule

(vi) Six Central cabinet ministers, including the home minister, to be nominated by the Prime
Minister.

Five Ministers of Cabinet rank / Minister of State (independent charge) nominated by the
Chairman of the Council (i.e., Prime Minister) are permanent invitees to the Council.

There is also a Standing Committee of the Council. It was set up in 1996 for
continuous consultation and processing of matters for the consideration of the
Council. It consists of the following members:

(i) Union Home Minister as the Chairman. Hence, option (c) is correct.

(ii) Five Union Cabinet Ministers

(iii) Nine Chief Ministers

The Council is assisted by a secretariat called the Inter-State Council Secretariat.


This secretariat was set-up in 1991 and is headed by a secretary to the Government of
India. Since 2011, it is also functioning as the secretariat of the Zonal Councils.

22. Which of the following factors have been taken into account to form zones in Zonal council?
1. The natural divisions of the country
2. The river systems and means of communication
3. The cultural and linguistic affinity
4. The requirements of economic development
5. The security and law and order

Select the correct answer using the code given below:

www.insightsactivelearn.com 18
Total Marks : 200
Test-24 (Subject)
( INSTA Prelims Test Series 2021 )

A. 1, 2 and 4 only
B. 2, 3 and 4 only
C. 2, 3 and 5 only
D. 1, 2, 3, 4 and 5

Correct Answer : D

Answer Justification :

All the above statements are correct.

The Zonal Councils are the statutory (and not the constitutional) bodies. They are
established by an Act of the Parliament, that is, States Reorganisation Act of 1956.

The act divided the country into five zones (Northern, Central, Eastern, Western and
Southern) and provided a zonal council for each zone.

While forming these zones, several factors have been taken into account which
include: the natural divisions of the country, the river systems and means of
communication, the cultural and linguistic affinity and the requirements of economic
development, security and law and order.

Each zonal council consists of the following members: (a) home minister of Central
government. (b) chief ministers of all the States in the zone. (c) Two other ministers from
each state in the zone. (d) Administrator of each union territory in the zone.

23. Consider the following statements regarding National emergency


1. The proclamation of Emergency must be approved by Parliament within two months.
2. The emergency, once approved by the Parliament, could remain in operation as long as the
Executive (cabinet) desired.
3. The resolution approving the proclamation of emergency must be passed by Parliament by a
special majority.

Which of the statements given above is/are correct?


A. 1 only
B. 2 and 3 only
C. 3 only
D. 1, 2 and 3

Correct Answer : C

Answer Justification :

National emergency

www.insightsactivelearn.com 19
Total Marks : 200
Test-24 (Subject)
( INSTA Prelims Test Series 2021 )

Parliamentary Approval and Duration

The proclamation of Emergency must be approved by both the Houses of Parliament


within one month from the date of its issue. Hence, statement 1 is incorrect.

Originally, the period allowed for approval by the Parliament was two months, but was
reduced by the 44th Amendment Act of 1978. However, if the proclamation of emergency is
issued at a time when the Lok Sabha has been dissolved or the dissolution of the Lok Sabha
takes place during the period of one month without approving the proclamation, then the
proclamation survives until 30 days from the first sitting of the Lok Sabha after its
reconstitution, provided the Rajya Sabha has in the meantime approved it.

If approved by both the Houses of Parliament, the emergency continues for six months, and
can be extended to an indefinite period with an approval of the Parliament for every six
months.

This provision for periodical parliamentary approval was also added by the 44th
Amendment Act of 1978. Before that, the emergency, once approved by the
Parliament, could remain in operation as long as the Executive (cabinet) desired.
Hence, statement 2 is incorrect.

Every resolution approving the proclamation of emergency or its continuance must be


passed by either House of Parliament by a special majority, that is,

(a) a majority of the total membership of that house, and (b) a majority of not less than two-
thirds of the members of that house present and voting. This special majority provision was
introduced by the 44th Amendment Act of 1978. Previously, such resolution could be passed
by a simple majority of the Parliament.

24. The President of India is entitled to which of the following privileges and immunities?
1. He enjoys personal immunity from legal liability for his official acts.
2. During his term of office, he is immune from any criminal proceedings, except those of his
personal acts.
3. He cannot be arrested or imprisoned.

Which of the statements given above are correct?


A. 1 and 2 only
B. 2 and 3 only
C. 1 and 3 only
D. 1, 2 and 3

Correct Answer : C

Answer Justification :

www.insightsactivelearn.com 20
Total Marks : 200
Test-24 (Subject)
( INSTA Prelims Test Series 2021 )

Conditions of President’s Office

The Constitution lays down the following conditions of the President’s office:

1. He should not be a member of either House of Parliament or a House of the state


legislature. If any such person is elected as President, he is deemed to have vacated his seat in
that House on the date on which he enters upon his office as President.

2. He should not hold any other office of profit.

3. He is entitled, without payment of rent, to the use of his official residence (the Rastrapathi
Bhavan).

4. He is entitled to such emoluments, allowances and privileges as may be determined by


Parliament.

5. His emoluments and allowances cannot be diminished during his term of office.

The President is entitled to a number of privileges and immunities. He enjoys personal


immunity from legal liability for his official acts. During his term of office, he is
immune from any criminal proceedings, even in respect of his personal acts. He
cannot be arrested or imprisoned. Hence, statement 2 is incorrect.

However, after giving two months’ notice, civil proceedings can be instituted against him
during his term of office in respect of his personal acts.

25. Consider the following statements regarding veto power of Indian President
1. When Absolute Veto is exercised by President, the bill ends and does not become an act.
2. The President possess Suspensive Veto in the case of money bills.
3. The President has no veto power in respect of a constitutional amendment bill.

Which of the statements given above are correct?


A. 1 and 2 only
B. 2 and 3 only
C. 1 and 3 only
D. 1, 2 and 3

Correct Answer : C

Answer Justification :

The President of India is vested with three–absolute veto, suspensive veto and pocket veto.
There is no qualified veto in the case of Indian President; it is possessed by the American
President.

Absolute Veto

www.insightsactivelearn.com 21
Total Marks : 200
Test-24 (Subject)
( INSTA Prelims Test Series 2021 )

It refers to the power of the President to withhold his assent to a bill passed by the
Parliament. The bill then ends and does not become an act.

Suspensive Veto

The President exercises this veto when he returns a bill for reconsideration of the Parliament.
However, if the bill is passed again by the Parliament with or without amendments and again
presented to the President, it is obligatory for the President to give his assent to the bill.

The President does not possess this veto in the case of money bills. Hence, statement
2 is incorrect.

The President can either give his assent to a money bill or withhold his assent to a money bill
but cannot return it for the reconsideration of the Parliament.

Pocket Veto

In this case, the President neither ratifies nor rejects nor returns the bill, but simply keeps the
bill pending for an indefinite period.

It should be noted here that the President has no veto power in respect of a
constitutional amendment bill. The 24th Constitutional Amendment Act of 1971 made it
obligatory for the President to give his assent to a constitutional amendment bill.

26. Consider the following statements regarding Prime Minister


1. Under the Indian Constitution, the term of the Prime Minister is five years and holds office
during the pleasure of the president.
2. Prime Minister gets the salary and allowances that are payable to a member of Parliament.

Which of the statements given above is/are correct?


A. 1 only
B. 2 only
C. Both 1 and 2
D. Neither 1 nor 2

Correct Answer : B

Answer Justification :

OATH, TERM AND SALARY

Before the Prime Minister enters upon his office, the president administers to him the
oaths of office and secrecy.

www.insightsactivelearn.com 22
Total Marks : 200
Test-24 (Subject)
( INSTA Prelims Test Series 2021 )

The term of the Prime Minister is not fixed and he holds office during the pleasure of
the president. Hence, statement 1 is incorrect.

However, this does not mean that the president can dismiss the Prime Minister at any time. So
long as the Prime Minister enjoys the majority support in the Lok Sabha, he cannot be
dismissed by the President. However, if he loses the confidence of the Lok Sabha, he must
resign or the President can dismiss him.

The salary and allowances of the Prime Minister are determined by the Parliament from time
to time. He gets the salary and allowances that are payable to a member of Parliament.

Since the Prime Minister stands at the head of the council of ministers, the other ministers
cannot function when the Prime Minister resigns or dies. In other words, the resignation or
death of an incumbent Prime Minister automatically dissolves the council of ministers and
thereby generates a vacuum.

27. Which of the following statements are related to Council of Ministers


1. Article 74 deals with the appointment, tenure, responsibility, qualification, oath and salaries
and allowances of the ministers.
2. The total number of ministers, excluding the Prime Minister, in the Council of Ministers shall
not exceed 15% of the total strength of the Lok Sabha.
3. A minister who is not a member of the Parliament (either house) for any period of six
consecutive months shall cease to be a minister.

Which of the statements given above is/are correct?


A. 1 only
B. 2 and 3 only
C. 3 only
D. 1, 2 and 3

Correct Answer : C

Answer Justification :

Council of Ministers

The principles of parliamentary system of government are not detailed in the Constitution, but
two Articles (74 and 75) deal with them in a broad, sketchy and general manner. Article 74
deals with the status of the council of ministers while Article 75 deals with the
appointment, tenure, responsibility, qualification, oath and salaries and allowances of
the ministers. Hence, statement 1 is incorrect.

Article 75–Other Provisions as to Ministers

1. The Prime Minister shall be appointed by the President and the other Ministers shall be

www.insightsactivelearn.com 23
Total Marks : 200
Test-24 (Subject)
( INSTA Prelims Test Series 2021 )

appointed by the President on the advice of the Prime Minister.

2. The total number of ministers, including the Prime Minister, in the Council of
Ministers shall not exceed 15% of the total strength of the Lok Sabha. Hence,
statement 2 is incorrect.

This provision was added by the 91st Amendment Act of 2003.

3. A member of either house of Parliament belonging to any political party who is disqualified
on the ground of defection shall also be disqualified to be appointed as a minister. This
provision was also added by the 91st Amendment Act of 2003.

4. The ministers shall hold office during the pleasure of the President.

5. The council of ministers shall be collectively responsible to the Lok Sabha.

6. The President shall administer the oaths of office and secrecy to a minister.

7. A minister who is not a member of the Parliament (either house) for any period of
six consecutive months shall cease to be a minister.

8. The salaries and allowances of ministers shall be determined by the Parliament.

28. Consider the following statements regarding Rajya Sabha


1. The Constitution has fixed six years the term of office of members of the Rajya Sabha.
2. The Rajya Sabha was first constituted in 1950.

Which of the statements given above is/are correct?


A. 1 only
B. 2 only
C. Both 1 and 2
D. Neither 1 nor 2

Correct Answer : D

Answer Justification :

Duration of Rajya Sabha

The Rajya Sabha (first constituted in 1952) is a continuing chamber, that is, it is a
permanent body and not subject to dissolution. Hence, statement 2 is incorrect.

However, one-third of its members retire every second year. Their seats are filled up by fresh
elections and presidential nominations at the beginning of every third year. The retiring
members are eligible for re-election and renomination any number of times.

www.insightsactivelearn.com 24
Total Marks : 200
Test-24 (Subject)
( INSTA Prelims Test Series 2021 )

The Constitution has not fixed the term of office of members of the Rajya Sabha and
left it to the Parliament. Hence, statement 1 is incorrect.

Accordingly, the Parliament in the Representation of the People Act (1951) provided that the
term of office of a member of the Rajya Sabha shall be six years. The act also empowered the
president of India to curtail the term of members chosen in the first Rajya Sabha. In the first
batch, it was decided by lottery as to who should retire. Further, the act also authorised the
President to make provisions to govern the order of retirement of the members of the Rajya
Sabha.

29. Who among the following has been provided with statutory recognition?

A. Leader of the House of Parliament


B. Leader of the Opposition
C. Whip
D. None of the above

Correct Answer : B

Answer Justification :

Leader of the House

Under the Rules of Lok Sabha, the ‘Leader of the House’ means the prime minister, if
he is a member of the Lok Sabha, or a minister who is a member of the Lok Sabha and
is nominated by the prime minister to function as the Leader of the House. There is
also a ‘Leader of the House’ in the Rajya Sabha. He is a minister and a member of the
Rajya Sabha and is nominated by the prime minister to function as such.

Leader of the Opposition

In each House of Parliament, there is the ‘Leader of the Opposition’. The leader of the largest
Opposition party having not less than one-tenth seats of the total strength of the House is
recognised as the leader of the Opposition in that House. In a parliamentary system of
government, the leader of the opposition has a significant role to play. His main functions are
to provide a

constructive criticism of the policies of the government and to provide an alternative


government. Therefore, the leader of Opposition in the Lok Sabha and the Rajya Sabha
were accorded statutory recognition in 1977. Hence, option (b) is correct.

Whip

Though the offices of the leader of the House and the leader of the Opposition are not
mentioned in the Constitution of India, they are mentioned in the Rules of the House and

www.insightsactivelearn.com 25
Total Marks : 200
Test-24 (Subject)
( INSTA Prelims Test Series 2021 )

Parliamentary Statute respectively. The office of ‘whip’, on the other hand, is mentioned
neither in the Constitution of India nor in the Rules of the House nor in a Parliamentary
Statute. It is based on the conventions of the parliamentary government.

30. Which of the following are the consequences of Prorogation?


1. It not only terminates a sitting but also a session of the House.
2. All pending notices lapse on prorogation.
3. Prorogation brings to an end all bills pending before the House.

Which of the statements given above is/are correct?


A. 1 only
B. 2 and 3 only
C. 3 only
D. 1, 2 and 3

Correct Answer : A

Answer Justification :

Prorogation

1. It not only terminates a sitting but also a session of the House.

2. It is done by the president of India.

3. It also does not affect the bills or any other business pending before the House. Hence,
statement 3 is incorrect.

However, all pending notices (other than those for introducing bills) lapse on
prorogation and fresh notices have to be given for the next session. Hence, statement
2 is incorrect.

In Britain, prorogation brings to an end all bills or any other business pending before
the House.

31. Consider the following statements regarding Zero Hour


1. It is mentioned in the Rules of Procedure.
2. It is an Indian innovation in the field of parliamentary procedures.
3. The zero hour starts immediately after the question hour and lasts until the agenda for the day
is taken up.

Which of the statements given above is/are correct?


A. 1 only

www.insightsactivelearn.com 26
Total Marks : 200
Test-24 (Subject)
( INSTA Prelims Test Series 2021 )

B. 2 and 3 only
C. 3 only
D. 1, 2 and 3

Correct Answer : B

Answer Justification :

Zero Hour

Unlike the question hour, the zero hour is not mentioned in the Rules of Procedure.
Hence, statement 1 is incorrect.

Thus, it is an informal device available to the members of the Parliament to raise matters
without any prior notice. The zero hour starts immediately after the question hour and
lasts until the agenda for the day (i.e., regular business of the House) is taken up. In
other words, the time gap between the question hour and the agenda is known as zero hour. It
is an Indian innovation in the field of parliamentary procedures and has been in
existence since 1962.

32. Which of the following are the restrictions on Adjournment Motion?


1. It should raise a matter which is definite, factual, urgent and of public importance.
2. It should not cover more than one matter.
3. It should not raise a question of privilege.

Which of the statements given above is/are correct?


A. 1 only
B. 2 and 3 only
C. 3 only
D. 1, 2 and 3

Correct Answer : D

Answer Justification :

All the above statements are correct.

Adjournment Motion

www.insightsactivelearn.com 27
Total Marks : 200
Test-24 (Subject)
( INSTA Prelims Test Series 2021 )

It is introduced in the Parliament to draw attention of the House to a definite matter of urgent
public importance, and needs the support of 50 members to be admitted. As it interrupts the
normal business of the House, it is regarded as an extraordinary device. It involves an element
of censure against the government and hence Rajya Sabha is not permitted to make use of this
device. The discussion on an adjournment motion should last for not less than two hours and
thirty minutes. The right to move a motion for an adjournment of the business of the House is
subject to the following restrictions:

1. It should raise a matter which is definite, factual, urgent and of public importance;

2. It should not cover more than one matter;

3. It should be restricted to a specific matter of recent occurrence and should not be framed in
general terms;

4. It should not raise a question of privilege;

5. It should not revive discussion on a matter that has been discussed in the same session;

6. It should not deal with any matter that is under adjudication by court; and

7. It should not raise any question that can be raised on a distinct motion.

33. Consider the following statements regarding Youth Parliament


1. The scheme of Youth Parliament was started on the recommendation of the First All India
Whips Conference.
2. The ministry of parliamentary affairs provides necessary training and encouragement to the
states in introducing the scheme.

Which of the statements given above is/are correct?


A. 1 only
B. 2 only
C. Both 1 and 2
D. Neither 1 nor 2

Correct Answer : B

Answer Justification :

Youth Parliament

The scheme of Youth Parliament was started on the recommendation of the Fourth
All India Whips Conference. Hence, statement 1 is incorrect.

www.insightsactivelearn.com 28
Total Marks : 200
Test-24 (Subject)
( INSTA Prelims Test Series 2021 )

Its objectives are:

1. to acquaint the younger generations with practices and procedures of Parliament;

2. to imbibe the spirit of discipline and tolerance cultivating character in the minds of youth;
and

3. to inculcate in the student community the basic values of democracy and to enable them to
acquire a proper perspective on the functioning of democratic institutions.

The ministry of parliamentary affairs provides necessary training and encouragement


to the states in introducing the scheme.

34. Consider the following statements regarding Public Bill


1. It is introduced in the Parliament by a member of ruling party.
2. Its introduction in the House does not requires any notice period.
3. It reflects of the policies of the government (ruling party).

Which of the statements given above is/are correct?


A. 1 only
B. 2 and 3 only
C. 3 only
D. 1, 2 and 3

Correct Answer : C

Answer Justification :

Public Bill

1. It is introduced in the Parliament by a minister. Hence, statement 1 is incorrect.

2. It reflects of the policies of the government (ruling party).

3. It has greater chance to be approved by the Parliament.

4. Its rejection by the House amounts to the expression of want of parliamentary confidence in
the government and may lead to its resignation.

5. Its introduction in the House requires seven days’ notice. Hence, statement 2 is
incorrect.

6. It is drafted by the concerned department in consultation with the law department.

www.insightsactivelearn.com 29
Total Marks : 200
Test-24 (Subject)
( INSTA Prelims Test Series 2021 )

35. Consider the following statements regarding Money Bills


1. A money bill can only be introduced in the Lok Sabha.
2. Every such bill is considered to be a government bill and can be introduced only by a minister.
3. The Rajya Sabha can reject or amend a money bill.

Which of the statements given above are correct?


A. 1 and 2 only
B. 2 and 3 only
C. 1 and 3 only
D. 1, 2 and 3

Correct Answer : A

Answer Justification :

Money Bills

Article 110 of the Constitution deals with the definition of money bills. It states that a bill is
deemed to be a money bill if it contains ‘only’ provisions dealing with all or any of the matters
mentioned under article 110.

If any question arises whether a bill is a money bill or not, the decision of the Speaker of the
Lok Sabha is final. His decision in this regard cannot be questioned in any court of law or in
the either House of Parliament or even the president. When a money bill is transmitted to the
Rajya Sabha for recommendation and presented to the president for assent, the Speaker
endorses it as a money bill.

The Constitution lays down a special procedure for the passing of money bills in the
Parliament. A money bill can only be introduced in the Lok Sabha and that too on the
recommendation of the president. Every such bill is considered to be a government bill
and can be introduced only by a minister.

After a money bill is passed by the Lok Sabha, it is transmitted to the Rajya Sabha for its
consideration. The Rajya Sabha has restricted powers with regard to a money bill. It
cannot reject or amend a money bill. Hence, statement 3 is incorrect.

It can only make the recommendations. It must return the bill to the Lok Sabha within 14 days,
whether with or without recommendations. The Lok Sabha can either accept or reject all or
any of the recommendations of the Rajya Sabha. If the Rajya Sabha does not return the bill to
the Lok Sabha within 14 days, the bill is deemed to have been passed by both the Houses in
the form originally passed by the Lok Sabha.

36. Consider the following statements regarding Railway Budget


1. The Railway Budget was separated from the General Budget in 1921.

www.insightsactivelearn.com 30
Total Marks : 200
Test-24 (Subject)
( INSTA Prelims Test Series 2021 )

2. It was separated on the recommendations of the Acworth Committee Report.

Which of the statements given above is/are correct?


A. 1 only
B. 2 only
C. Both 1 and 2
D. Neither 1 nor 2

Correct Answer : B

Answer Justification :

The Railway Budget was separated from the General Budget in 1924 on the
recommendations of the Acworth Committee Report (1921). Hence, statement 1 is
incorrect.

The reasons or objectives of this separation were as follows:

1. To introduce flexibility in railway finance.

2. To facilitate a business approach to the railway policy.

3. To secure stability of the general revenues by providing an assured annual contribution


from railway revenues.

4. To enable the railways to keep their profits for their own development (after paying a fixed
annual contribution to the general revenues).

In 2017, the Central Government merged the railway budget into the general budget. Hence,
there is now only one budget for the Government of India i.e., Union Budget.

37. Consider the following statements regarding Panel of Vice-Chairpersons of Rajya Sabha
1. Under the Rules of Rajya Sabha, the Deputy Chairman nominates from amongst the members
a panel of vice-chairpersons.
2. Any member of panel can preside over the House in the absence of the Chairman or the
Deputy Chairman.

Which of the statements given above is/are correct?


A. 1 only
B. 2 only
C. Both 1 and 2
D. Neither 1 nor 2

Correct Answer : B

www.insightsactivelearn.com 31
Total Marks : 200
Test-24 (Subject)
( INSTA Prelims Test Series 2021 )

Answer Justification :

Panel of Vice-Chairpersons of Rajya Sabha

Under the Rules of Rajya Sabha, the Chairman nominates from amongst the members
a panel of vice-chairpersons. Hence, statement 1 is incorrect.

Any one of them can preside over the House in the absence of the Chairman or the
Deputy Chairman. He has the same powers as the Chairman when so presiding. He holds
office until a new panel of vice-chairpersons is nominated.

When a member of the panel of vice chairpersons is also not present, any other person as
determined by the House acts as the Chairman. It must be emphasized here that a member of
the panel of vice chairpersons cannot preside over the House, when the office of the Chairman
or the Deputy Chairman is vacant. During such time, the Chairman’s duties are to be
performed by such member of the House as the president may appoint for the purpose. The
elections are held, as soon as possible, to fill the vacant posts.

38. Consider the following statements regarding Public Accounts Committee


1. This committee was first set up under the provisions of the Government of India Act of 1935.
2. The chairman of the committee is generally selected from the Opposition.

Which of the statements given above is/are correct?


A. 1 only
B. 2 only
C. Both 1 and 2
D. Neither 1 nor 2

Correct Answer : B

Answer Justification :

Public Accounts Committee

This committee was set up first in 1921 under the provisions of the Government of
India Act of 1919 and has since been in existence. Hence, statement 1 is incorrect.

At present, it consists of 22 members (15 from the Lok Sabha and 7 from the Rajya Sabha).
The members are elected by the Parliament every year from amongst its members according
to the principle of proportional representation by means of the single transferable vote. Thus,
all parties get due representation in it. The term of office of the members is one year. A
minister cannot be elected as a member of the committee. The chairman of the committee is
appointed from amongst its members by the Speaker. Until 1966 - ‘67, the chairman of the
committee belonged to the ruling party. However, since 1967 a convention has developed
whereby the chairman of the committee is selected invariably from the Opposition.

www.insightsactivelearn.com 32
Total Marks : 200
Test-24 (Subject)
( INSTA Prelims Test Series 2021 )

39. Consider the following statements regarding Committee on Papers Laid on the Table
1. This committee was constituted in 1953.
2. The Committee consists of 22 members, 15 from the Lok Sabha and 7 from the Rajya Sabha.
3. It examines all papers laid on the table of the House by ministers to see whether they comply
with provisions of the Constitution.

Which of the statements given above is/are correct?


A. 1 only
B. 2 and 3 only
C. 3 only
D. 1, 2 and 3

Correct Answer : C

Answer Justification :

Committee on Papers Laid on the Table

This committee was constituted in 1975. Hence, statement 1 is incorrect.

The Lok Sabha Committee has 15 members, while the Rajya Sabha Committee has 10
members. Hence, statement 2 is incorrect.

It examines all papers laid on the table of the House by ministers to see whether they
comply with provisions of the Constitution, or the related Act or Rule. It does not examine
statutory notifications and orders that fall under the jurisdiction of the Committee on
Subordinate Legislation.

40. Under the constitution, the President can seek the opinion on which of the following matters from Supreme
Court?
1. On any question of law or fact of public importance which is likely to arise.
2. On any dispute arising out of any pre-constitution treaty.
3. On any dispute arising out of any agreement or covenant.

Which of the statements given above is/are correct?


A. 1 only
B. 2 and 3 only
C. 3 only
D. 1, 2 and 3

Correct Answer : D

Answer Justification :

www.insightsactivelearn.com 33
Total Marks : 200
Test-24 (Subject)
( INSTA Prelims Test Series 2021 )

All the above statements are correct.

Advisory Jurisdiction

The Constitution (Article 143) authorizes the president to seek the opinion of the Supreme
Court in the two categories of matters:

(a) On any question of law or fact of public importance which has arisen or which is
likely to arise.

(b) On any dispute arising out of any pre-constitution treaty, agreement, covenant,
engagement, sanad or other similar instruments.

In the first case, the Supreme Court may tender or may refuse to tender its opinion to the
president. But, in the second case, the Supreme Court ‘must’ tender its opinion to the
president. In both the cases, the opinion expressed by the Supreme Court is only advisory and
not a judicial pronouncement. Hence, it is not binding on the president; he may follow or may
not follow the opinion. However, it facilitates the government to have an authoritative legal
opinion on a matter to be decided by it. So far (2019), the President has made fifteen
references to the Supreme Court under its advisory jurisdiction (also known as consultative
jurisdiction).

41. Consider the following statements regarding legislative council


1. A resolution to abolish a legislative council must be passed by the state assembly by a special
majority.
2. The act of creating or abolishing legislative council is deemed as an amendment of the
Constitution for the purposes of Article 368.

Which of the statements given above is/are correct?


A. 1 only
B. 2 only
C. Both 1 and 2
D. Neither 1 nor 2

Correct Answer : A

Answer Justification :

The twenty-two states have unicameral system. Here, the state legislature consists of the
governor and the legislative assembly. In the states having bicameral system, the state
legislature consists of the governor, the legislative council and the legislative assembly. The
legislative council (Vidhan Parishad) is the upper house (second chamber or house of elders),
while the legislative assembly (Vidhan Sabha) is the lower house (first chamber or popular
house).

www.insightsactivelearn.com 34
Total Marks : 200
Test-24 (Subject)
( INSTA Prelims Test Series 2021 )

The Constitution provides for the abolition or creation of legislative councils in states.
Accordingly, the Parliament can abolish a legislative council (where it already exists) or create
it (where it does not exist), if the legislative assembly of the concerned state passes a
resolution to that effect. Such a specific resolution must be passed by the state
assembly by a special majority, that is, a majority of the total membership of the
assembly and a majority of not less than two-thirds of the members of the assembly
present and voting. This Act of Parliament is not to be deemed as an amendment of
the Constitution for the purposes of Article 368 and is passed like an ordinary piece
of legislation (ie, by simple majority). Hence, statement 2 is incorrect.

42. Consider the following statements regarding legislative assembly


1. Its maximum strength is fixed at 400 and minimum strength at 40.
2. Some members of the legislative assemblies in Sikkim and Nagaland are elected indirectly.

Which of the statements given above is/are correct?


A. 1 only
B. 2 only
C. Both 1 and 2
D. Neither 1 nor 2

Correct Answer : B

Answer Justification :

Composition of Assembly

Strength

The legislative assembly consists of representatives directly elected by the people on the basis
of universal adult franchise. Its maximum strength is fixed at 500 and minimum
strength at 60. Hence, statement 1 is incorrect.

It means that its strength varies from 60 to 500 depending on the population size of the state.
However, in case of Arunachal Pradesh, Sikkim and Goa, the minimum number is fixed at 30
and in case of Mizoram and Nagaland, it is 40 and 46 respectively. Further, some members
of the legislative assemblies in Sikkim and Nagaland are also elected indirectly.

43. Consider the following statements regarding legislative procedure under state legislature
1. The Constitution does not provide for the mechanism of joint sitting of two Houses of the state
legislature.
2. A deadlock between the two Houses takes place when the legislative council after receiving a
bill passed by the legislative assembly, does not pass the bill within six months.
3. If the President gives his assent to the bill reserved by Governor, it becomes an act only after

www.insightsactivelearn.com 35
Total Marks : 200
Test-24 (Subject)
( INSTA Prelims Test Series 2021 )

the assent of the Governor.

Which of the statements given above is/are correct?


A. 1 only
B. 2 and 3 only
C. 3 only
D. 1, 2 and 3

Correct Answer : A

Answer Justification :

A deadlock between the two Houses takes place when the legislative council, after
receiving a bill passed by the legislative assembly, rejects the bill or proposes
amendments that are not acceptable to the legislative assembly or does not pass the
bill within three months. Hence, statement 2 is incorrect.

The Constitution does not provide for the mechanism of joint sitting of two Houses of
the state legislature to resolve a deadlock between them over the passage of a bill.

The legislative assembly can override the legislative council by passing the bill for the second
time and not vice versa. When a bill is passed by the assembly for the second time and
transmitted to the legislative council, if the legislative council rejects the bill again, or
proposes amendments that are not acceptable to the legislative assembly, or does not pass the
bill within one month, then the bill is deemed to have been passed by both the Houses in the
form in which it was passed by the legislative assembly for the second time.

When the governor reserves a bill for the consideration of the President, he will not have any
further role in the enactment of the bill. If the bill is returned by the President for the
reconsideration of the House or Houses and is passed again, the bill must be presented again
for the presidential assent only. If the President gives his assent to the bill, it becomes
an act. This means that the assent of the Governor is no longer required. Hence,
statement 3 is incorrect.

44. Which of the following provisions made under constitution for independence of a high court?
1. The judges of a high court are appointed by the president in consultation with the members of
the judiciary itself.
2. The judges of a high court are provided with the security of tenure.
3. The salaries, allowances, privileges, leave and pension of the judges of a high court are
determined from time to time by the state legislature.
4. The salaries, allowances and pensions of the judges are charged on the consolidated fund of
the state.

Select the correct answer using the code given below:


A. 1 and 2 only

www.insightsactivelearn.com 36
Total Marks : 200
Test-24 (Subject)
( INSTA Prelims Test Series 2021 )

B. 3 and 4 only
C. 2, 3 and 4 only
D. 1, 2, 3 and 4

Correct Answer : A

Answer Justification :

The independence of a high court is very essential for the effective discharge of the duties
assigned to it.

1. Mode of Appointment

The judges of a high court are appointed by the president (which means the cabinet)
in consultation with the members of the judiciary itself.

2. Security of Tenure

The judges of a high court are provided with the security of tenure. They can be
removed from office by the president only in the manner and on the grounds mentioned in the
Constitution.

3. Fixed Service Conditions

The salaries, allowances, privileges, leave and pension of the judges of a high court
are determined from time to time by the Parliament.

4. Expenses Charged on Consolidated Fund

The salaries and allowances of the judges, the salaries, allowances and pensions of
the staff as well as the administrative expenses of a high court are charged on the
consolidated fund of the state. Thus, they are non-votable by the state legislature
(though they can be discussed by it). It should be noted here that the pension of a
high court judge is charged on the Consolidated Fund of India and not the state.

5. Ban on Practice after Retirement

The retired permanent judges of a high court are prohibited from pleading or acting in any
court or before any authority in India except the Supreme Court and the other high courts.

6. Freedom to Appoint its Staff

The chief justice of a high court can appoint officers and servants of the high court without any
interference from the executive. He can also prescribe their conditions of service.

www.insightsactivelearn.com 37
Total Marks : 200
Test-24 (Subject)
( INSTA Prelims Test Series 2021 )

45. Consider the following statements regarding State Administrative Tribunals


1. The Administrative Tribunals Act of 1985 empowers the State government to establish the
State Administrative Tribunals (SATs).
2. The chairman and members of the SATs are appointed by the Governor.

Which of the statements given above is/are correct?


A. 1 only
B. 2 only
C. Both 1 and 2
D. Neither 1 nor 2

Correct Answer : D

Answer Justification :

State Administrative Tribunals

The Administrative Tribunals Act of 1985 empowers the Central government to


establish the State Administrative Tribunals (SATs) on specific request of the concerned
state governments. Hence, statement 1 is incorrect.

So far (2019), the SATs have been set up in the nine states of Andhra Pradesh, Himachal
Pradesh, Odisha, Karnataka, Madhya Pradesh, Maharashtra, Tamil Nadu, West Bengal and
Kerala. However, the Madhya Pradesh, Tamil Nadu and Himachal Pradesh Tribunals have
since been abolished. But subsequently, the Himachal Pradesh reestablished the SAT and the
state of Tamil Nadu has also requested now to reestablish the same. Further, the state
government of Haryana has requested to establish the SAT for their state. On the other hand,
the state government of Odisha has submitted a proposal for abolition of Odisha
Administrative Tribunal.

Like the CAT, the SATs exercise original jurisdiction in relation to recruitment and all service
matters of state government employees.

The chairman and members of the SATs are appointed by the president after
consultation with the governor of the state concerned. Hence, statement 2 is
incorrect.

46. Consider the following statements regarding Legal Services in India


1. The National Legal Services Authority (NALSA) has been constituted under the Legal Services
Authorities Act, 1987.
2. The State Legal Services Authority is headed by the Chief Justice of the respective High Court.
3. The National Legal Services Authority (NALSA) organizes Lok Adalats for amicable settlement
of disputes.

www.insightsactivelearn.com 38
Total Marks : 200
Test-24 (Subject)
( INSTA Prelims Test Series 2021 )

Which of the statements given above is/are correct?


A. 1 only
B. 2 and 3 only
C. 3 only
D. 1, 2 and 3

Correct Answer : D

Answer Justification :

All the above statements are correct.

Article 39A of the Constitution of India provides for free legal aid to the poor and weaker
sections of the society and ensures justice for all.

The National Legal Services Authority (NALSA) has been constituted under the Legal
Services Authorities Act, 1987 to provide free Legal Services to the weaker sections of
the society and to organize Lok Adalats for amicable settlement of disputes.

Hon’ble Mr. Justice Sharad Arvind Bobde, The Chief Justice of India is the Patron-in-
Chief and Hon’ble Mr. Justice N. V. Ramana, Judge, Supreme Court of India is the
Executive Chairman of the Authority. NALSA is housed at 12/11, Jam Nagar House, New
Delhi-110011.

In every State, State Legal Services Authority has been constituted to give effect to the
policies and directions of the NALSA and to give free legal services to the people and conduct
Lok Adalats in the State. The State Legal Services Authority is headed by Hon’ble the
Chief Justice of the respective High Court who is the Patron-in-Chief of the State Legal
Services Authority.

In every District, District Legal Services Authority has been constituted to implement Legal
Services Programmes in the District. The District Legal Services Authority is situated in the
District Courts Complex in every District and chaired by the District Judge of the respective
district.

47. Consider the following statements regarding special provisions for the Hyderabad-Karnataka region
1. The special provisions for the Hyderabad-Karnataka region of the state of Karnataka were
provided under the 98th Constitutional Amendment Act of 2012.
2. President have the responsibility to establishment a separate development board for
Hyderabad-Karnataka region.

Which of the statements given above is/are correct?


A. 1 only
B. 2 only
C. Both 1 and 2

www.insightsactivelearn.com 39
Total Marks : 200
Test-24 (Subject)
( INSTA Prelims Test Series 2021 )

D. Neither 1 nor 2

Correct Answer : C

Answer Justification :

All the above statements are correct.

Under Article 371-J, the President is empowered to provide that the Governor of
Karnataka would have special responsibility for

1. The establishment of a separate development board for Hyderabad-Karnataka


region.

2. Making a provision that a report on the working of the board would be placed every year
before the State Legislative Assembly.

3. The equitable allocation of funds for developmental expenditure over the region

4. The reservation of seats in educational and vocational training institutions in the region for
students who belong to the region

5. The reservation in state government posts in the region for persons who belong to the
region

Article 371-J (which provided for special provisions for the Hyderabad-Karnataka
region of the state of Karnataka) was inserted in the Constitution by the 98th
Constitutional Amendment Act of 2012. The special provisions aim to establish an
institutional mechanism for equitable allocation of funds to meet the development needs over
the region, as well as to enhance human resources and promote employment from the region
by providing for local cadres in service and reservation in educational and vocational training
institutions.

48. Which of the following States or Areas are exempted under 73rd amendment act of 1992?
1. Nagaland
2. Meghalaya
3. Mizoram
4. Darjeeling district of West Bengal

Select the correct answer using the code given below:


A. 1, 2 and 3 only
B. 2, 3 and 4 only
C. 1, 2 and 4 only
D. 1, 2, 3 and 4

www.insightsactivelearn.com 40
Total Marks : 200
Test-24 (Subject)
( INSTA Prelims Test Series 2021 )

Correct Answer : D

Answer Justification :

All the above statements are correct.

73rd amendment act of 1992

Significance of the Act

This act has added a new Part-IX to the Constitution of India. This part is entitled as ‘The
Panchayats’ and consists of provisions from Articles 243 to 243 O. In addition, the act has also
added a new Eleventh Schedule to the Constitution. This schedule contains 29 functional items
of the panchayats. It deals with Article 243-G.

Exempted States and Areas

The act does not apply to the states of Nagaland, Meghalaya and Mizoram and certain
other areas. These areas include, (a) the scheduled areas and the tribal areas in the
states; (b) the hill areas of Manipur for which district councils exist; and (c)
Darjeeling district of West Bengal for which Darjeeling Gorkha Hill Council exists.

However, the Parliament may extend the provisions of this Part to the scheduled areas and
tribal areas subject to such exceptions and modifications as it may specify. Under this
provision, the Parliament has enacted the “Provisions of the Panchayats (Extension to the
Scheduled Areas Act”, 1996, popularly known as the PESA Act or the Extension Act.

49. Consider the following statements regarding District Planning Committee


1. All the members of a district planning committee should be elected by the elected members of
the panchayat and municipalities of that district.
2. Under 73rd amendment act of 1992, District Planning Committee shall be constituted in every
State at the district level.

Which of the statements given above is/are correct?


A. 1 only
B. 2 only
C. Both 1 and 2
D. Neither 1 nor 2

Correct Answer : D

Answer Justification :

Statement of Objects and Reasons appended to the Constitution (Seventy-third Amendment)

www.insightsactivelearn.com 41
Total Marks : 200
Test-24 (Subject)
( INSTA Prelims Test Series 2021 )

Bill, 1991 which was enacted as the Constitution (Seventy-fourth Amendment) Act, 1992.
Hence, statement 2 is incorrect.

https://www.india.gov.in/my-government/constitution-india/amendments/constitution-india-sev
enty-fourth-amendment-act-1992

District Planning Committee

District Planning Committee (DPC) is the committee created as per article 243ZD of the
Constitution of India at the district level for planning at the district.

Committee for district planning.-(1) There shall be constituted in every State at the
district level a District Planning Committee to consolidate the plans prepared by the
Panchayats and the Municipalitiies in the district and to prepare a draft development plan for
the district as a whole.

The state legislature may make provisions with respect to the following:

1. The composition of such committees;

2. The manner of election of members of such committees;

3. The functions of such committees in relation to district planning; and

4. The manner of the election of the chairpersons of such committees.

The act lays down that four-fifths of the members of a district planning committee
should be elected by the elected members of the district panchayat and
municipalities in the district from amongst themselves. Hence, statement 1 is
incorrect.

The representation of these members in the committee should be in proportion to the ratio
between the rural and urban populations in the district.

The chairperson of such committee shall forward the development plan to the state
government.

In preparing the draft development plan, a district planning committee shall

(a) Have regard to–

(i) matters of common interest between the Panchayats and Municipalities including spatial
planning, sharing of water other physical and natural resources, the integrated develop of
infrastructure and environmental conservation;

(ii) the extent and type of available resources whether financial otherwise; and

(b) Consult such institutions and organizations as the Governor may specify.

www.insightsactivelearn.com 42
Total Marks : 200
Test-24 (Subject)
( INSTA Prelims Test Series 2021 )

50. Consider the following statements regarding Municipal Corporation


1. They are established in the states by the acts of the concerned state legislatures.
2. The Mayor is responsible for the implementation of the decisions taken by the council.
3. Mayor is elected in a majority of the states for a term co-terminus with the term of Council.

Which of the statements given above is/are correct?


A. 1 only
B. 2 and 3 only
C. 3 only
D. 1, 2 and 3

Correct Answer : A

Answer Justification :

Municipal Corporation

Municipal corporations are created for the administration of big cities like Delhi, Mumbai,
Kolkata, Hyderabad, Bangalore and others. They are established in the states by the acts
of the concerned state legislatures, and in the union territories by the acts of the
Parliament of India. There may be one common act for all the municipal corporations in a
state or a separate act for each municipal corporation.

A municipal corporation has three authorities, namely, the council, the standing committees
and the commissioner. The Council is the deliberative and legislative wing of the corporation.
It consists of the Councillors directly elected by the people, as well as a few nominated
persons having knowledge or experience of municipal administration. In brief, the composition
of the Council including the reservation of seats for SCs, STs and women is governed by the
74th Constitutional Amendment Act. The Council is headed by a Mayor. He is assisted by
a Deputy Mayor. He is elected in a majority of the states for a one-year renewable
term. Hence, statement 3 is incorrect.

He is basically an ornamental figure and a formal head of the corporation. His main function is
to preside over the meetings of the Council.

The municipal commissioner is responsible for the implementation of the decisions


taken by the council and its standing committees. Hence, statement 2 is incorrect.

Thus, he is the chief executive authority of the corporation. He is appointed by the state
government and is generally a member of the IAS.

51. Which of the following are functions and powers of district councils under Sixth Schedule?
1. They can make laws on matters like land, forests, canal water, shifting cultivation and village
www.insightsactivelearn.com 43
Total Marks : 200
Test-24 (Subject)
( INSTA Prelims Test Series 2021 )

administration.
2. They are empowered to assess and collect land revenue.
3. They can establish, construct or manage a primary school.

Which of the statements given above is/are correct?


A. 1 only
B. 2 and 3 only
C. 3 only
D. 1, 2 and 3

Correct Answer : D

Answer Justification :

All the above statements are correct.

The various features of administration contained in the Sixth Schedule are as follows:

1. The district and regional councils administer the areas under their jurisdiction. They can
make laws on certain specified matters like land, forests, canal water, shifting
cultivation, village administration, inheritance of property, marriage and divorce,
social customs and so on. But all such laws require the assent of the governor.

2. The district and regional councils within their territorial jurisdictions can constitute village
councils or courts for trial of suits and cases between the tribes. They hear appeals from them.
The jurisdiction of high court over these suits and cases is specified by the governor.

3. The district council can establish, construct or manage primary schools,


dispensaries, markets, ferries, fisheries, roads and so on in the district. It can also make
regulations for the control of money lending and trading by non-tribals. But such regulations
require the assent of the governor.

4. The district and regional councils are empowered to assess and collect land
revenue and to impose certain specified taxes.

52. Which of the following are powers and functions of the Election Commission of India?
1. To prepare and periodically revise electoral rolls and to register all eligible voters.
2. To advise the president on matters relating to the disqualifications of the members of
Parliament.
3. To act as a court for settling disputes related to granting of recognition to political parties.
4. To advise the governor on matters relating to the disqualifications of the members of state
legislature.

Select the correct answer using the code given below:


A. 1, 2 and 3 only

www.insightsactivelearn.com 44
Total Marks : 200
Test-24 (Subject)
( INSTA Prelims Test Series 2021 )

B. 2, 3 and 4 only
C. 1, 2 and 4 only
D. 1, 2, 3 and 4

Correct Answer : D

Answer Justification :

All the above statements are correct.

The powers and functions of the Election Commission with regard to elections to the
Parliament, state legislatures and offices of President and Vice-President can be classified into
three categories, viz,

1. Administrative

2. Advisory

3. Quasi-Judicial

In detail, these powers and functions are:

1. To determine the territorial areas of the electoral constituencies throughout the country on
the basis of the Delimitation Commission Act of Parliament.

2. To prepare and periodically revise electoral rolls and to register all eligible voters.

3. To notify the dates and schedules of elections and to scrutinize nomination papers.

4. To grant recognition to political parties and allot election symbols to them.

5. To act as a court for settling disputes related to granting of recognition to political parties
and allotment of election symbols to them.

6. To appoint officers for inquiring into disputes relating to electoral arrangements.

7. To determine the code of conduct to be observed by the parties and the candidates at the
time of elections.

8. To prepare a roster for publicity of the policies of the political parties on radio and TV in
times of elections.

9. To advise the president on matters relating to the disqualifications of the members of


Parliament.

10. To advise the governor on matters relating to the disqualifications of the members of state
legislature.

11. To cancel polls in the event of rigging, booth capturing, violence and other irregularities.

www.insightsactivelearn.com 45
Total Marks : 200
Test-24 (Subject)
( INSTA Prelims Test Series 2021 )

53. Consider the following statements regarding Finance Commission


1. The Constitution provides for the qualifications of members of the commission and the manner
in which they should be selected.
2. The Finance Commission consists of a chairman and four other members to be appointed by
the president.
3. The chairman and members hold office for such period as specified by the president in his
order.

Which of the statements given above is/are correct?


A. 1 only
B. 2 and 3 only
C. 3 only
D. 1, 2 and 3

Correct Answer : B

Answer Justification :

The Finance Commission consists of a chairman and four other members to be


appointed by the president. They hold office for such period as specified by the
president in his order. They are eligible for reappointment.
The Constitution authorizes the Parliament to determine the qualifications of
members of the commission and the manner in which they should be selected. Hence,
statement 1 is incorrect.

Accordingly, the Parliament has specified the qualifications of the chairman and members of
the commission.

The chairman should be a person having experience in public affairs and the four other
members should be selected from amongst the following:

1. A judge of high court or one qualified to be appointed as one.

2. A person who has specialized knowledge of finance and accounts of the government.

3. A person who has wide experience in financial matters and in administration.

4. A person who has special knowledge of economics.

54. Consider the following statements regarding CAG


1. The accountability of the executive to the Parliament in the sphere of financial administration
is secured through audit reports of the CAG.
www.insightsactivelearn.com 46
Total Marks : 200
Test-24 (Subject)
( INSTA Prelims Test Series 2021 )

2. The CAG is an agent of the Parliament and conducts audit of expenditure on behalf of the
Parliament.
3. CAG is the only body in India which has the auditing role of the secret service expenditure.

Which of the statements given above are correct?


A. 1 and 2 only
B. 2 and 3 only
C. 1 and 3 only
D. 1, 2 and 3

Correct Answer : A

Answer Justification :

The role of CAG is to uphold the Constitution of India and the laws of Parliament in the field of
financial administration. The accountability of the executive (i.e., council of ministers)
to the Parliament in the sphere of financial administration is secured through audit
reports of the CAG. The CAG is an agent of the Parliament and conducts audit of
expenditure on behalf of the Parliament. Therefore, he is responsible only to the
Parliament.

The secret service expenditure is a limitation on the auditing role of the CAG. Hence,
statement 3 is incorrect.

In this regard, the CAG cannot call for particulars of expenditure incurred by the executive
agencies, but has to accept a certificate from the competent administrative authority that the
expenditure has been so incurred under his authority.

55. Who among the following is the chairman of high-powered selection committee to make recommendations for
appointment of Members in CAT?

A. Chief Justice of India


B. Sitting Judge of Supreme Court
C. Prime Minister
D. Cabinet Secretary

Correct Answer : B

Answer Justification :

The Central Administrative Tribunal (CAT) was set up in 1985 with the principal bench at
Delhi and additional benches in different states. At present, it has 17 regular benches, 15 of
which operate at the principal seats of high courts and the remaining two at Jaipur and
Lucknow. These benches also hold circuit sittings at other seats of high courts.

www.insightsactivelearn.com 47
Total Marks : 200
Test-24 (Subject)
( INSTA Prelims Test Series 2021 )

The CAT is a multi-member body consisting of a chairman and members. The appointment of
Members in CAT is made on the basis of recommendations of a high-powered
selection committee chaired by a sitting Judge of Supreme Court who is nominated by
the Chief Justice of India. Hence, option (b) is correct.

After obtaining the concurrence of Chief Justice of India, appointments are made with the
approval of Appointments Committee of the Cabinet (ACC).

56. Consider the following statements regarding Law Commission of India


1. The Commission was originally constituted in 1955 and is re-constituted every three years.
2. The Law Commission of India is a statutory body constituted by the Government of India.
3. Secretary, Department of Legal Affairs is the ex-officio Chairperson.

Which of the statements given above is/are correct?


A. 1 only
B. 2 and 3 only
C. 3 only
D. 1, 2 and 3

Correct Answer : A

Answer Justification :

Law Commission of India

Background

The Law Commission of India is a non-statutory body constituted by the Government


of India from time to time. Hence, statement 2 is incorrect.

The Commission was originally constituted in 1955 and is re-constituted every three
years. The tenure of twenty-first Law Commission of India was upto 31st August, 2018.

The various Law Commission have been able to make important contribution towards the
progressive development and codification of Law of the country. The Law Commission has so
far submitted 277 reports.

The 22nd Law Commission will be constituted for a period of three years from the date of
publication of its Order in the Official Gazette. It will consist of:

1. a full-time Chairperson;

2. four full-time Members (including Member-Secretary)

www.insightsactivelearn.com 48
Total Marks : 200
Test-24 (Subject)
( INSTA Prelims Test Series 2021 )

3. Secretary, Department of Legal Affairs as ex-officio Member; Hence, statement 3


is incorrect.

4. Secretary, Legislative Department as ex officio Member; and

5. not more than five part-time Members.

The Union Cabinet, chaired by the Prime Minister, Shri Narendra Modi has approved Twenty-
second Law Commission of India for a period of three years from the date of publication of the
Order of Constitution in the Official Gazette.

Benefits

The Government will have the benefit of recommendations from a specialised body on
different aspects of law which are entrusted to the Commission for its study and
recommendations, as per its terms of reference.

The Law Commission shall, on a reference made to it by the Central Government or suo-motu,
undertake research in law and review of existing laws in India for making reforms therein and
enacting new legislations. It shall also undertake studies and research for bringing reforms in
the justice delivery systems for elimination of delay in procedures, speedy disposal of cases,
reduction in cost of litigation etc.

57. Who among the following are members of select committee for National Human Rights Commission?
1. Prime minister
2. Chairman of the Rajya Sabha
3. Speaker of the Lok Sabha
4. Opposition in both the Houses of Parliament

Select the correct answer using the code given below:


A. 1, 2 and 3 only
B. 2, 3 and 4 only
C. 1, 2 and 4 only
D. 1, 3 and 4 only

Correct Answer : D

Answer Justification :

The National Human Rights Commission is a statutory (and not a constitutional) body. It was
established in 1993 under a legislation enacted by the Parliament, namely, the Protection of
Human Rights Act, 1993.

www.insightsactivelearn.com 49
Total Marks : 200
Test-24 (Subject)
( INSTA Prelims Test Series 2021 )

The chairperson and members are appointed by the president on the


recommendations of a six-member committee consisting of the prime minister as its
head, the Speaker of the Lok Sabha, the Deputy Chairman of the Rajya Sabha, leaders
of the Opposition in both the Houses of Parliament and the Central home minister.
Hence, option (d) is correct.

Further, a sitting judge of the Supreme Court or a sitting chief justice of a high court can be
appointed only after consultation with the chief justice of India.

58. Consider the following statements regarding State disaster management authority
1. An SDMA consists of a chairperson and other members, not exceeding five.
2. The Home Minister of the state is the ex-officio chairperson of the SDMA.

Which of the statements given above is/are correct?


A. 1 only
B. 2 only
C. Both 1 and 2
D. Neither 1 nor 2

Correct Answer : D

Answer Justification :

State disaster management authority

Composition

Every state government should establish a State Disaster Management Authority (SDMA) for
the state. An SDMA consists of a chairperson and other members, not exceeding nine.
The Chief Minister of the state is the ex-officio chairperson of the SDMA. The
chairperson of the State Executive Committee is the ex-officio member of the SDMA.
Hence, both statement 1 and 2 are incorrect.

The other members, not exceeding eight, are nominated by the chairperson of the SDMA. The
chairperson of the SDMA designates one of the members as the vice-chairperson of the SDMA.
The chairperson of the State Executive Committee acts as the ex-officio chief executive officer
of the SDMA.

59. Which of the following is/are the constitutional provisions with respect to Language of the union?
1. Hindi written in Devanagari script is to be the official language of the Union.
2. After fifteen years of commencement of the Constitution, the Parliament may provide for the
continued use of English language for the specified purposes.

www.insightsactivelearn.com 50
Total Marks : 200
Test-24 (Subject)
( INSTA Prelims Test Series 2021 )

Which of the statements given above is/are correct?


A. 1 only
B. 2 only
C. Both 1 and 2
D. Neither 1 nor 2

Correct Answer : C

Answer Justification :

All the above statements are correct.

Language of the union

The Constitution contains the following provisions in respect of the official language of the
Union.

1. Hindi written in Devanagari script is to be the official language of the Union. But,
the form of numerals to be used for the official purposes of the Union has to be the
international form of Indian numerals and not the Devanagari form of numerals.

2. However, for a period of fifteen years from the commencement of the Constitution (i.e., from
1950 to 1965), the English language would continue to be used for all the official purposes of
the Union for which it was being used before 1950.

3. Even after fifteen years, the Parliament may provide for the continued use of
English language for the specified purposes.

4. At the end of five years, and again at the end of ten years, from the commencement of the
Constitution, the president should appoint a commission to make recommendations with
regard to the progressive use of the Hindi language, restrictions on the use of the English
language and other related issues.

5. A committee of Parliament is to be constituted to examine the recommendations of the


commission and to report its views on them to the president.

Accordingly, in 1955, the president appointed an Official Language Commission under the
chairmanship of B.G. Kher. The commission submitted its report to the President in 1956. The
report was examined by a committee of Parliament constituted in 1957 under the
chairmanship of Gobind Ballabh Pant. However, another Official Language Commission (as
envisaged by the Constitution) was not appointed in 1960.

60. Consider the following statements regarding All-India Services


1. Under Article 312, the Parliament can create new all India services, excluding an all-India

www.insightsactivelearn.com 51
Total Marks : 200
Test-24 (Subject)
( INSTA Prelims Test Series 2021 )

judicial service.
2. The 44nd Amendment Act of 1976 made the provision for the creation of all India judicial
service.

Which of the statements given above is/are correct?


A. 1 only
B. 2 only
C. Both 1 and 2
D. Neither 1 nor 2

Correct Answer : D

Answer Justification :

All-India Services

Article 312 makes the following provisions in respect of all-India services:

(a) The Parliament can create new all India services (including an all-India judicial
service), if the Rajya Sabha passes a resolution declaring that it is necessary or
expedient in the national interest to do so. Hence, statement 1 is incorrect.

Such a resolution in the Rajya Sabha should be supported by two-thirds of the members
present and voting. This power of recommendation is given to the Rajya Sabha to protect the
interests of states in the Indian federal system.

(b) Parliament can regulate the recruitment and conditions of service of persons appointed to
all-India services. Accordingly, the Parliament has enacted the All-India Services Act, 1951 for
the purpose.

(c) The services known at the commencement of the Constitution (that is, January 26, 1950) as
the Indian Administrative Service and the Indian Police Service are deemed to be services
created by Parliament under this provision.

(d) The all-India judicial service should not include any post inferior to that of a district judge.
A law providing for the creation of this service is not to be deemed as an amendment of the
Constitution for the purposes of Article 368.

Though the 42nd Amendment Act of 1976 made the provision for the creation of all
India judicial service, no such law has been made so far.

Hence, both statement 1 and 2 are incorrect.

61. Consider the following statements regarding Anti-Defection Law


1. Any question regarding disqualification arising out of defection is to be decided by the
presiding officer of the House.

www.insightsactivelearn.com 52
Total Marks : 200
Test-24 (Subject)
( INSTA Prelims Test Series 2021 )

2. The Parliament is empowered to make rules to give effect to the provisions of the Tenth
Schedule.
3. According to the rules, the presiding officer can suo-motu take up a defection case.

Which of the statements given above is/are correct?


A. 1 only
B. 2 and 3 only
C. 3 only
D. 1, 2 and 3

Correct Answer : A

Answer Justification :

Anti-Defection Law

Deciding Authority

Any question regarding disqualification arising out of defection is to be decided by


the presiding officer of the House. Originally, the act provided that the decision of the
presiding officer is final and cannot be questioned in any court. However, in Kihoto Hollohan
case (1993), the Supreme Court declared this provision as unconstitutional on the ground that
it seeks to take away the jurisdiction of the Supreme Court and the high courts. It held that the
presiding officer, while deciding a question under the Tenth Schedule, function as a tribunal.
Hence, his decision like that of any other tribunal, is subject to judicial review on the grounds
of mala fides, perversity, etc. But, the court rejected the contention that the vesting of
adjudicatory powers in the presiding officer is by itself invalid on the ground of political bias.

Rule-Making Power

The presiding officer of a House is empowered to make rules to give effect to the
provisions of the Tenth Schedule. Hence, statement 2 is incorrect.

All such rules must be placed before the House for 30 days. The House may approve or modify
or disapprove them. Further, he may direct that any willful contravention by any member of
such rules may be dealt with in the same manner as a breach of privilege of the House.

According to the rules made so, the presiding officer can take up a defection case
only when he receives a complaint from a member of the House. Hence, statement 3
is incorrect.

Before taking the final decision, he must give the member (against whom the complaint has
been made) a chance to submit his explanation. He may also refer the matter to the committee
of privileges for inquiry. Hence, defection has no immediate and automatic effect.

62. Consider the following statements regarding Asian Infrastructure Investment Bank

www.insightsactivelearn.com 53
Total Marks : 200
Test-24 (Subject)
( INSTA Prelims Test Series 2021 )

1. It is headquartered in Shanghai.
2. All the powers of the AIIB are vested in the Board of Directors.
3. The Board of Directors are composed of twelve members.

Which of the statements given above is/are correct?


A. 1 only
B. 2 and 3 only
C. 3 only
D. 1, 2 and 3

Correct Answer : C

Answer Justification :

Asian Infrastructure Investment Bank

The Asian Infrastructure Investment Bank (AIIB) is a multilateral development bank with
a mission to improve social and economic outcomes in Asia.

It is established by the AIIB Articles of Agreement (entered into force Dec. 25, 2015)
which is a multilateral treaty. The Parties (57 founding members) to agreement
comprise the Membership of the Bank.

It is headquartered in Beijing and began its operations in January 2016. Hence,


statement 1 is incorrect.

The members to Bank have now grown to 97 approved members worldwide. There
are 27 prospective members including Armenia, Lebanon, Brazil, South Africa,
Greece, etc.

Fourteen of the G-20 nations are AIIB members including France, Germany, Italy and
the United Kingdom.

By investing in sustainable infrastructure and other productive sectors in Asia and


beyond, it will better connect people, services and markets that over time will impact
the lives of billions and build a better future.

Governance

Board of Governors

www.insightsactivelearn.com 54
Total Marks : 200
Test-24 (Subject)
( INSTA Prelims Test Series 2021 )

The Board of Governors consists of one Governor and one Alternate Governor
appointed by each member country.

Governors and Alternate Governors serve at the pleasure of the appointing


member. All the powers of the AIIB are vested in the Board of Governors.
Hence, statement 2 is incorrect.

Board of Directors

The Board of Directors are composed of twelve members who shall not be members of
the Board of Governors, and of whom:

nine are elected by the Governors representing regional members; and

three are elected by the Governors representing non-regional members.

63. Consider the following statements regarding World Intellectual Property Organization
1. WIPO was created in 1967 to encourage creative activity, to promote the protection of
intellectual property throughout the world.
2. India is a founding member of WIPO.
3. All member states of the UN are entitled, though not obliged, can become members of WIPO.

Which of the statements given above are correct?


A. 1 and 2 only
B. 2 and 3 only
C. 1 and 3 only
D. 1, 2 and 3

Correct Answer : C

Answer Justification :

World Intellectual Property Organisation (WIPO)


WIPO was created in 1967 "to encourage creative activity, to promote the
protection of intellectual property throughout the world".
WIPO currently administers 26 international treaties.
It is headquartered in Geneva, Switzerland.

www.insightsactivelearn.com 55
Total Marks : 200
Test-24 (Subject)
( INSTA Prelims Test Series 2021 )

Membership

WIPO currently has 191 member states.

All member states of the UN are entitled, though not obliged, to become
members of the specialized agencies like WIPO.

188 of UN member states as well as Cook Islands, Holy See and Niue are members of
WIPO.

Palestine has permanent observer status.

Some 250 non-governmental organizations (NGOs) and intergovernmental organizations


(IGOs) have official observer status at WIPO meetings.

India joined WIPO in 1975. Hence, statement 2 is incorrect.

64. Consider the following statements regarding European Union


1. The European Union is a group of 28 countries that operate as a cohesive economic and
political block.
2. Schengen Agreement paved the way for the creation of open borders without passport
controls.
3. A monetary union was established in 1999 and all EU member states use the euro currency.

Which of the statements given above are correct?


A. 1 and 2 only
B. 2 and 3 only
C. 1 and 3 only
D. 1, 2 and 3

Correct Answer : A

Answer Justification :

European Union
The European Union is a group of 28 countries that operate as a cohesive economic
and political block.

19 of these countries use EURO as their official currency. 9 EU members (Bulgaria, Croatia,

www.insightsactivelearn.com 56
Total Marks : 200
Test-24 (Subject)
( INSTA Prelims Test Series 2021 )

Czech Republic, Denmark, Hungary, Poland, Romania, Sweden, and the United Kingdom) do
not use the euro.

The EU grew out of a desire to form a single European political entity to end centuries of
warfare among European countries that culminated with World War II and decimated much of
the continent.

The EU has developed an internal single market through a standardised system of laws that
apply in all member states in matters, where members have agreed to act as one.

Schengen Agreement (1985) paved the way for the creation of open borders without
passport controls between most member states. It was effective in 1995.

A monetary union was established in 1999 and came into full force in 2002 and is
composed of 19 EU member states which use the euro currency. Hence, statement 3
is incorrect.

These are Austria, Belgium, Cyprus, Estonia, Finland, France, Germany, Greece, Ireland, Italy,
Latvia, Lithuania, Luxembourg, Malta, Netherlands, Portugal, Slovakia, Slovenia, and Spain.

65. Consider the following statements regarding International Development Association (IDA)
1. The International Development Association (IDA) is the part of the IMF that helps the world’s
poorest countries.
2. IDA lends money on concessional terms i.e., at zero or very low interest charge and
repayments are stretched over 30 to 40 years.

Which of the statements given above is/are correct?


A. 1 only
B. 2 only
C. Both 1 and 2
D. Neither 1 nor 2

Correct Answer : B

Answer Justification :

The International Development Association (IDA) is the part of the World Bank that
helps the world’s poorest countries. Hence, statement 1 is incorrect.

Overseen by 173 shareholder nations, IDA aims to reduce poverty by providing zero to low-
interest loans (called “credits”) and grants for programs that boost economic growth, reduce
inequalities, and improve people’s living conditions.

IDA complements the World Bank’s original lending arm—the International Bank for
Reconstruction and Development (IBRD). IBRD was established to function as a self-sustaining

www.insightsactivelearn.com 57
Total Marks : 200
Test-24 (Subject)
( INSTA Prelims Test Series 2021 )

business and provides loans and advice to middle-income and credit-worthy poor countries.
IBRD and IDA share the same staff and headquarters and evaluate projects with the same
rigorous standards.

IDA is one of the largest sources of assistance for the world’s 74 poorest countries and is the
single largest source of donor funds for basic social services in these countries. IDA lends
money on concessional terms. This means that IDA credits have a zero or very low
interest charge and repayments are stretched over 30 to 40 years, including a 5- to
10-year grace period. IDA also provides grants to countries at risk of debt distress.

66. Consider the following statements regarding Asia-Europe Meeting (ASEM)


1. The Asia-Europe Meeting (ASEM) is an intergovernmental process established in 1996 to
foster dialogue and cooperation between Asia and Europe.
2. India is a member to this forum.

Which of the statements given above is/are correct?


A. 1 only
B. 2 only
C. Both 1 and 2
D. Neither 1 nor 2

Correct Answer : C

Answer Justification :

All the above statements are correct.

Overview

The Asia-Europe Meeting (ASEM) is an intergovernmental process established in


1996 to foster dialogue and cooperation between Asia and Europe.

ASEM addresses political, economic, financial, social, cultural, and educational issues of
common interest in a spirit of mutual respect and equal partnership. Its foremost event, the
ASEM Summit, is a biennial meeting between the Heads of State and Government, the
President of the European Council, the President of the European Commission, and the
Secretary-General of ASEAN. In addition, ASEM Ministers & Senior Officials also meet in their
respective sectoral dialogues.

The initial ASEM Partnership in 1996 consisted of 15 EU Member States, 7 ASEAN Member
States, China, Japan, Korea and the European Commission. Today, ASEM comprises 53
Partners: 30 European and 21 Asian countries, the European Union and the ASEAN
Secretariat.

Through its informal process based on equal partnership and enhancing mutual

www.insightsactivelearn.com 58
Total Marks : 200
Test-24 (Subject)
( INSTA Prelims Test Series 2021 )

understanding, ASEM facilitates and stimulates progress but does not seek to duplicate
bilateral and other multilateral relationships between Asia and Europe.

India joined Asia-Europe Meeting (ASEM) in 2008.

67. Consider the following statements regarding Uniting for Consensus


1. The Uniting for Consensus is a group of 12 countries, also referred to as the Coffee Club.
2. It opposes expansion of UNSC non-permanent membership.

Which of the statements given above is/are correct?


A. 1 only
B. 2 only
C. Both 1 and 2
D. Neither 1 nor 2

Correct Answer : A

Answer Justification :

The Uniting for Consensus group of 12 countries, also referred to as the Coffee Club,
proposes expanding UNSC non-permanent membership from 10 to 20 members, and
various changes to the workings of the Council. Hence, statement 2 is incorrect.

1. The Security Council shall consist of twenty-five Members of the United Nations. France,
the People’s Republic of China, the Russian Federation, the United Kingdom of Great
Britain and Northern Ireland and the United States of America shall be permanent
members of the Security Council. The General Assembly shall elect twenty other
Members of the United Nations to be non-permanent members of the Security Council,
due regard being specially paid, in the first instance to the contribution of Members of
the United Nations to the maintenance of international peace and security and to the
other purposes of the Organization, and also to equitable geographical distribution.

2. The non-permanent members of the Security Council shall be elected for a term of two
years. In the first election of the non-permanent members after the increase of the
membership of the Security Council from fifteen to twenty-five, five of the retiring
members shall continue for one more year. Non-permanent members may be eligible for
immediate re-election, subject to the decision of their respective geographical groups.

https://electthecouncil.org/timeline-post/the-coffee-club-proposal/

https://www.un.org/press/en/2005/ga10371.doc.htm

www.insightsactivelearn.com 59
Total Marks : 200
Test-24 (Subject)
( INSTA Prelims Test Series 2021 )

68. Consider the following statements regarding IBSA


1. It was established in 2003.
2. IBSA is a coordinating mechanism amongst three emerging countries, three multi ethnic and
multicultural democracies.
3. The headquarters and permanent executive secretariat of IBSA is located in South Africa.

Which of the statements given above are correct?


A. 1 and 2 only
B. 2 and 3 only
C. 1 and 3 only
D. 1, 2 and 3

Correct Answer : A

Answer Justification :

IBSA

THE INDIA-BRAZIL-SOUTH AFRICA DIALOGUE FORUM

Established in June 2003, IBSA is a coordinating mechanism amongst three emerging


countries, three multi ethnic and multicultural democracies, which are determined to:

contribute to the construction of a new international architecture

bring their voice together on global issues

deepen their ties in various areas

IBSA also opens itself to concrete projects of cooperation and partnership with less developed
countries.
The establishment of IBSA was formalized by the Brasilia Declaration of 6 June 2003, which
mentions India, Brazil and South Africa's democratic credentials, their condition as developing
nations and their capacity of acting on a global scale as the main reasons for the three
countries to come together. Their status as middle powers, their common need to address
social inequalities within their borders and the existence of consolidated industrial areas in the
three countries are often mentioned as additional elements that bring convergence among the
members of the Forum.

The principles, norms and values underpinning the IBSA Dialogue Forum are participatory
democracy, respect for human rights and the Rule of Law. The strength of IBSA is the shared
vision of the three countries that democracy and development are mutually reinforcing and
key to sustainable peace and stability.

www.insightsactivelearn.com 60
Total Marks : 200
Test-24 (Subject)
( INSTA Prelims Test Series 2021 )

IBSA keeps an open and flexible structure. IBSA does not have a headquarters or a
permanent executive secretariat. Hence, statement 3 is incorrect.

69. Consider the following statements regarding Nuclear Suppliers Group (NSG)
1. It is a group of nuclear supplier countries that seeks to contribute to the non-proliferation of
nuclear weapons.
2. The NSG Guidelines are implemented by each Participating Government (PG) in accordance
with its national laws and practices.

Which of the statements given above is/are correct?


A. 1 only
B. 2 only
C. Both 1 and 2
D. Neither 1 nor 2

Correct Answer : C

Answer Justification :

All the above statements are correct.

The Nuclear Suppliers Group (NSG) is a group of nuclear supplier countries that
seeks to contribute to the non-proliferation of nuclear weapons through the
implementation of two sets of Guidelines for nuclear exports and nuclear-related exports.

The NSG Guidelines also contain the so-called “Non-Proliferation Principle,” adopted in 1994,
whereby a supplier, notwithstanding other provisions in the NSG Guidelines, authorizes a
transfer only when satisfied that the transfer would not contribute to the proliferation of
nuclear weapons. The Non-Proliferation Principle seeks to cover the rare but important cases
where adherence to the NPT or to a Nuclear Weapon Free Zone Treaty may not by itself be a
guarantee that a State will consistently share the objectives of the Treaty or that it will remain
in compliance with its Treaty obligations.

The NSG Guidelines are consistent with, and complement, the various international, legally
binding instruments in the field of nuclear non-proliferation. These include the Treaty on the
Non-Proliferation of Nuclear Weapons (NPT), the Treaty for the Prohibition of Nuclear
Weapons in Latin America (Treaty of Tlatelolco), the South Pacific Nuclear-Free-Zone Treaty
(Treaty of Rarotonga), the African Nuclear-Weapon-Free Zone Treaty (Treaty of Pelindaba),
the Treaty on the Southeast Asia Nuclear-Weapon-Free Zone (Treaty of Bangkok), and the
Central Asian Nuclear-Weapon-Free Zone Treaty (Treaty of Semipalatinsk).

The NSG Guidelines are implemented by each Participating Government (PG) in


accordance with its national laws and practices.
Decisions on export applications are taken at the national level in accordance with national
export licensing requirements.

www.insightsactivelearn.com 61
Total Marks : 200
Test-24 (Subject)
( INSTA Prelims Test Series 2021 )

https://www.nuclearsuppliersgroup.org/en/about-nsg

70. Who among the following are members of South Asia Subregional Economic Cooperation (SASEC)?
1. Bangladesh
2. Pakistan
3. Maldives
4. Myanmar

Select the correct answer using the code given below:


A. 1, 2 and 3 only
B. 2, 3 and 4 only
C. 1, 2 and 4 only
D. 1, 3 and 4 only

Correct Answer : D

Answer Justification :

The South Asia Subregional Economic Cooperation (SASEC) program brings together
Bangladesh, Bhutan, India, Maldives, Myanmar, Nepal, and Sri Lanka in a project-
based partnership that aims to promote regional prosperity, improve economic
opportunities, and build a better quality of life for the people of the subregion.
Hence, option (d) is correct.

SASEC countries share a common vision of boosting intraregional trade and cooperation in
South Asia, while also developing connectivity and trade with Southeast Asia through
Myanmar, to the People’s Republic of China, and the global market.

SASEC Vision

The SASEC Vision, and its supplement, SASEC Vision – Myanmar, articulate shared aspirations
of SASEC member countries, and set the path to achieve these through regional collaboration.
These documents lay out a plan to transform the subregion by leveraging natural resources,
promoting industry linkages for the development of regional value chains, and expanding the
region’s trade and commerce through the development of subregional gateways and hubs.

SASEC Operational Plan

The SASEC Operational Plan presents the strategic objectives of the SASEC partnership, and
the operational priorities of the four main SASEC sectors—transport, trade facilitation, energy,
and economic corridor development. It is supported by a list of potential projects regularly
updated by SASEC member countries to be implemented during 2016-2025.

https://www.sasec.asia/index.php?page=what-is-sasec

www.insightsactivelearn.com 62
Total Marks : 200
Test-24 (Subject)
( INSTA Prelims Test Series 2021 )

71. Consider the following statements regarding Supplementary Demands for Grants
1. Article 115 of Indian Constitution provides for Votes on account, votes of credit and
exceptional grants.
2. Article 116 of Indian Constitution provides for Supplementary, additional or excess grants.

Which of the statements given above is/are correct?


A. 1 only
B. 2 only
C. Both 1 and 2
D. Neither 1 nor 2

Correct Answer : D

Answer Justification :

Supplementary demand for grants:

Context:

The Lok Sabha has passed the supplementary demand for grants (second batch for 2020-21).

What are Supplementary Demands for Grants?

The supplementary demand for grants is needed for government expenditure over and above
the amount for which Parliamentary approval was already obtained during the Budget session.

Constitutional provisions:

Supplementary, additional or excess grants and Votes on account, votes of credit and
exceptional grants are mentioned in the Constitution of India 1949.

Article 115: Supplementary, additional or excess grants.

Article 116: Votes on account, votes of credit and exceptional grants.

Hence, both statement 1 and 2 are incorrect.

Procedure to be followed:

1. When grants, authorised by the Parliament, fall short of the required expenditure, an
estimate is presented before the Parliament for Supplementary or Additional grants.

2. These grants are presented and passed by the Parliament before the end of the financial

www.insightsactivelearn.com 63
Total Marks : 200
Test-24 (Subject)
( INSTA Prelims Test Series 2021 )

year.

3. When actual expenditure incurred exceeds the approved grants of the Parliament, the
Ministry of Finance presents a Demand for Excess Grant.

4. The Comptroller and Auditor General of India bring such excesses to the notice of the
Parliament.

5. The Public Accounts Committee examines these excesses and gives recommendations to
the Parliament.

6. The Demand for Excess Grants is made after the actual expenditure is incurred and is
presented to the Parliament after the end of the financial year in which the expenses
were made.

https://www.thehindu.com/news/national/parliament-proceedings-lok-sabha-passes-supplement
ary-demand-for-grants/article34101248.ece

72. Consider the following statements regarding Insurance Amendment Bill, 2021
1. The Bill increases the limit on foreign investment in an Indian insurance company from 49% to
100%.
2. Foreign investment in the insurance sector was first permitted in the year 2000 and up to
26%.

Which of the statements given above is/are correct?


A. 1 only
B. 2 only
C. Both 1 and 2
D. Neither 1 nor 2

Correct Answer : B

Answer Justification :

Insurance Amendment Bill, 2021:

Context:

The bill was recently passed by Rajya Sabha.

Key features of the Bill:

www.insightsactivelearn.com 64
Total Marks : 200
Test-24 (Subject)
( INSTA Prelims Test Series 2021 )

The Bill amends the Insurance Act, 1938 to increase the maximum foreign investment
allowed in an Indian insurance company.

The Bill increases the limit on foreign investment in an Indian insurance


company from 49% to 74%, and removes restrictions on ownership and control.
Hence, statement 1 is incorrect.

While control will go to foreign companies, the majority of directors and key
management persons will be resident Indians who will be covered by law of the land.

Significance:

Insurance companies are facing liquidity pressure and the higher limit would help meet the
growing capital requirement.

Background:

Foreign investment in the insurance sector was first permitted in the year 2000 up to
26%.

Subsequently, vide an Amendment Act of 2015, this limit was raised to 49% of the paid-
up equity capital of such company, which is Indian owned and controlled.

https://www.thehindu.com/news/national/parliament-proceedings-lok-sabha-passes-insurance-a
mendment-bill-2021to-raise-fdi-limit-to-74/article34132987.ece

73. Which of the following are the parameters used in Global Hunger Index (GHI)?

1. Undernourishment
2. Child Wasting
3. Child Stunting
4. Child Mortality

Select the correct answer using the code given below:


A. 1, 2 and 3 only
B. 2, 3 and 4 only
C. 1, 2 and 4 only
D. 1, 2, 3 and 4

Correct Answer : D

www.insightsactivelearn.com 65
Total Marks : 200
Test-24 (Subject)
( INSTA Prelims Test Series 2021 )

Answer Justification :

All the above statements are correct.

Government questions methodology and data accuracy of Global Hunger Index:

Context:

The government has questioned the methodology and data accuracy of the Global
Hunger Index (GHI) report, alleging that children considered healthy were also counted to
determine the ranking.

The government has already written to NGO Welthungerhilfe, which compiles the
report, expressing concerns about their methodology, data accuracy and sample size and
was yet to hear from them.

What’s the issue?

In the latest report, India was ranked below countries such as Nepal, Bangladesh and
Myanmar when it was among the top 10 food-producing countries in the world.

India was ranked at the 94th position out of 107 countries that were studied.

What is Global Hunger Index?

The report is a peer-reviewed publication released annually by Welthungerhilfe and Concern


Worldwide.

It tracks hunger at global, regional and national levels.

How are Countries ranked?

The GHI scores are based on a formula that captures three dimensions of hunger—insufficient
caloric intake, child undernutrition, and child mortality—using four component indicators:

UNDERNOURISHMENT: the share of the population that is under-nourished,


reflecting insufficient caloric intake

CHILD WASTING: the share of children under the age of five who are wasted (low
weight-for-height), reflecting acute undernutrition.

CHILD STUNTING: the share of children under the age of five who are stunted (low

www.insightsactivelearn.com 66
Total Marks : 200
Test-24 (Subject)
( INSTA Prelims Test Series 2021 )

height-for-age), reflecting chronic undernutrition.

CHILD MORTALITY: the mortality rate of children under the age of five.

https://www.thehindu.com/news/national/agriculture-mos-questions-global-hunger-index-repor
ts-methodology/article34107191.ece

74. Consider the following statements regarding aluminium-air batteries


1. Aluminium-air batteries utilise oxygen in the air to produce electricity.
2. Aluminium-air batteries can be recharged like lithium-ion batteries.
3. They are low cost and more energy-dense alternative to lithium-ion batteries.

Which of the statements given above are correct?


A. 1 and 2 only
B. 2 and 3 only
C. 1 and 3 only
D. 1, 2 and 3

Correct Answer : C

Answer Justification :

What are aluminium-air batteries?

Context:

State-owned Indian Oil Corporation Ltd. has entered into a joint venture with Israel-based
battery technology startup Phinergy to develop aluminium-air technology-based battery
systems for electric vehicles and stationary storage, as well as hydrogen storage solutions.

What is an aluminium-air battery?

Aluminium-air batteries utilise oxygen in the air which reacts with an aluminium hydroxide
solution to oxidise the aluminium and produce electricity.

Benefits:

Lower cost and more energy-dense alternative to lithium-ion batteries which are
currently in widespread use for electric vehicles in India.

Offer much greater range of 400 km or more per battery compared to lithium-ion
batteries which currently offer a range of 150-200 kilometres per full charge.

www.insightsactivelearn.com 67
Total Marks : 200
Test-24 (Subject)
( INSTA Prelims Test Series 2021 )

The aluminium plate in an aluminium-air battery is converted into aluminium


trihydroxide over time and that aluminium can be reclaimed from aluminium
trihydroxide or even traded directly for industrial uses.

Challenges:

Aluminium-air batteries cannot be recharged like lithium-ion batteries. Hence,


statement 2 is incorrect. Therefore, large scale use of aluminium-air battery based vehicles
would require the wide availability of battery swapping stations.

https://indianexpress.com/article/explained/indian-oil-corporation-phinergy-aluminium-batterie
s-7235560/

75. International Searching and International Preliminary Examining Authority (ISA/IPEA) is recognized by

A. World Health Organization


B. International Atomic Energy Agency
C. United Nations Educational, Scientific and Cultural Organization
D. World Intellectual Property Organization (WIPO)

Correct Answer : D

Answer Justification :

India, Japan agree for greater cooperation in patent verification:

Context:

India and Japan have agreed to recognize each other’s offices to act mutually as competent
International Searching and International Preliminary Examining Authority
(ISA/IPEA) for any international patent application filed with them.

This decision was taken during the recent review meet of Patent Prosecution Highway
(PPH) programme.

The Indian Patent Office was recognized as an International Searching Authority and
International Preliminary Examining Authority under the PCT and started functioning from
15th October 2013. We have access to a comprehensive collection of patent and non-patent
literature that covers the PCT minimum documentation. Professionally qualified and skilled
Examiners are our assets. Agreement was signed between the Indian Patent Office (IPO) and
the International Bureau (IB) of the World Intellectual Property Organization (WIPO).

ISA and IPEA Agreements

www.insightsactivelearn.com 68
Total Marks : 200
Test-24 (Subject)
( INSTA Prelims Test Series 2021 )

Agreements with the International Bureau of WIPO in relation to the functioning of the some
Authorities as International Searching and International Preliminary Examining Authorities.

Hence, option (d) is correct.

https://www.wipo.int/pct/en/access/isa_ipea_agreements.html

76. Consider the following statements regarding Gram UJALA


1. It is an ambitious scheme offering the world’s cheapest LED bulbs in rural areas at Rs.
2. The LED bulbs are offered by Convergence Energy Services Ltd (CESL).
3. The programme will be financed through carbon credits.

Which of the statements given above are correct?


A. 1 and 2 only
B. 2 and 3 only
C. 1 and 3 only
D. 1, 2 and 3

Correct Answer : B

Answer Justification :

Recently, the Government has launched the Gram UJALA Programme - an ambitious
scheme offering the world’s cheapest LED bulbs in rural areas at a mere Rs. 10.
Hence, statement 1 is incorrect.

Key Points

Coverage: In its first phase launched from Arrah in Bihar, 15 million LED bulbs will be
distributed across villages of 5 districts - Aarah (Bihar), Varanasi (Uttar Pradesh),
Vijayawada (Andhra Pradesh), Nagpur (Maharashtra), and villages in western Gujarat.

The light-emitting diode (LED) is one of today's most energy-efficient and rapidly-
developing lighting technologies.

Implementation:

7 watt and 12-watt LED bulbs with 3 years warranty will be given to rural consumers
against submission of working Incandescent bulbs.

www.insightsactivelearn.com 69
Total Marks : 200
Test-24 (Subject)
( INSTA Prelims Test Series 2021 )

Each household will get up to 5 LEDs.

Participating rural households will also have metres installed in their houses
to account for usage.

The LED bulbs are offered by state-run Energy Efficiency Services Ltd’s (EESL’s)
subsidiary Convergence Energy Services Ltd (CESL).

EESL is a Public Sector Undertaking (PSU) under the Ministry of Power.

Financing Mechanism:

The programme will be financed entirely through carbon credits and will be the first such
programme in India.

The revenue earned from carbon credits will contribute Rs. 60 per LED bulb piece, with
the balance Rs. 10 to be paid by the rural consumer.

https://www.thehindubusinessline.com/economy/policy/gram-ujala-scheme-to-save-power-and-c
ut-carbon-emission-considerably-rk-singh/article34150775.ece

77. Consider the following statements regarding Prithviraj Chauhan


1. Prithviraj Chauhan ruled under Tomar dynasty.
2. The Tomar dynasty shifted its capital in the 8th century, to Dhillikapuri (Delhi) during the
reign of Prithviraj Chauhan.

Which of the statements given above is/are correct?


A. 1 only
B. 2 only
C. Both 1 and 2
D. Neither 1 nor 2

Correct Answer : A

Answer Justification :

Tomar king Anangpal II:

Context:

The government has recently formed a committee to popularise the legacy of 11th-century
Tomar king, Anangpal II.

www.insightsactivelearn.com 70
Total Marks : 200
Test-24 (Subject)
( INSTA Prelims Test Series 2021 )

Who was Anangpal II?

1. Belonged to the Tomar dynasty that ruled parts of present-day Delhi and Haryana
between the 8th and 12th centuries.

2. Anangpal Tomar II was succeeded by his grandson Prithviraj Chauhan, who was
defeated by the Ghurid forces in the Battle of Tarain (present-day Haryana) after which
the Delhi Sultanate was established in 1192.

3. The Tomar dynasty shifted its capital in the 8th century), to Dhillikapuri (Delhi)
during the reign of Anangpal II. Hence, statement 2 is incorrect.

4. He gave Delhi its present name and also repopulated it.

5. He built Lal Kot fort and Anangtal Baoli.

https://indianexpress.com/article/explained/tomar-king-anangpal-ii-legacy-delhi-7237182/

78. Consider the following statements regarding United Nations Forum on Forests (UNFF)
1. In 2017, the Economic and Social Council of the United Nations (ECOSOC) established the
United Nations Forum on Forests (UNFF).
2. The Forum has universal membership, and is composed of all Member States of the United
Nations and specialized agencies.

Which of the statements given above is/are correct?


A. 1 only
B. 2 only
C. Both 1 and 2
D. Neither 1 nor 2

Correct Answer : B

Answer Justification :

United Nations Forum on Forests (UNFF)

Background

On 27 April 2017, the UN General Assembly adopted the first ever UN Strategic Plan for
Forests 2017-2030. The Strategic Plan provides a global framework for actions at all levels to
sustainably manage all types of forests and trees outside forests and halt deforestation and

www.insightsactivelearn.com 71
Total Marks : 200
Test-24 (Subject)
( INSTA Prelims Test Series 2021 )

forest degradation.At the heart of the Strategic Plan are six Global Forest Goals and 26
associated targets to be achieved by 2030, which are voluntary and universal. They support
the objectives of the International Arrangement on Forests and aim to contribute to progress
on the Sustainable Development Goals, the Aichi Biodiversity Targets, the Paris Agreement
adopted under the UN Framework Convention on Climate Change and other international
forest-related instruments, processes, commitments and goals.

As outlined in the Quadrennial programme of work of the Forum for the period 2017-2020,
under the Forum’s new format – odd-year sessions will focus on discussions on
implementation, technical advice and exchange of experiences while even-year sessions will
focus on policy dialogue, development and decision-making.

In October 2000, the Economic and Social Council of the United Nations (ECOSOC),
in its Resolution 2000/35 established the United Nations Forum on Forests (UNFF), a
subsidiary body with the main objective to promote “… the management, conservation
and sustainable development of all types of forests and to strengthen long-term
political commitment to this end…” based on the Rio Declaration, the Forest
Principles, Chapter 11 of Agenda 21 and the outcome of the IPF/IFF Processes and
other key milestones of international forest policy. Hence, statement 1 is incorrect.

The Forum has universal membership, and is composed of all Member States of the United
Nations and specialized agencies.

https://www.un.org/esa/forests/forum/about-unff/index.html

79. Consider the following statements regarding Ken and Betwa rivers
1. Ken rivers originate in MP and Betwa rivers originate in UP.
2. Ken and Betwa rivers are the tributaries of Yamuna.

Which of the statements given above is/are correct?


A. 1 only
B. 2 only
C. Both 1 and 2
D. Neither 1 nor 2

Correct Answer : B

Answer Justification :

About Ken- Betwa project:

Conceived as a two-part project, this is the country’s first river interlinking project.

It is perceived as a model plan for similar interstate river transfer missions.

www.insightsactivelearn.com 72
Total Marks : 200
Test-24 (Subject)
( INSTA Prelims Test Series 2021 )

The project aims to transfer surplus water from the Ken river in MP to Betwa in
UP to irrigate the drought-prone Bundelkhand region spread across the districts of
two states mainly Jhansi, Banda, Lalitpur and Mahoba districts of UP and Tikamgarh,
Panna and Chhatarpur districts of MP.

Key facts:

1. Ken and Betwa rivers originate in MP and are the tributaries of Yamuna. Hence,
statement 1 is incorrect.

2. Ken meets with Yamuna in Banda district of UP and with Betwa in Hamirpur district of
UP.

3. Rajghat, Paricha and Matatila dams are over Betwa river.

4. Ken River passes through Panna tiger reserve.

https://www.hindustantimes.com/india-news/explained-ken-betwa-river-interlinking-project-an
d-environmental-concerns-101616479775802.html

80. Jaapi, xorai and gamosa are traditional art and craft of

A. Assam
B. Sikkim
C. Meghalaya
D. Tripura

Correct Answer : A

Answer Justification :

The significance of jaapi, xorai and gamosa in Assam poll battle. Hence, option (a) is
correct.

Jaapi:

It is a conical hat made of bamboo and covered with dried tokou (a palm tree found in
rainforests of Upper Assam) leaves. While it is most often used in official functions to felicitate
guests, the landscape of rural Assam features a more utilitarian version, which farmers wear
to protect themselves from the harsh weather, both sun and rain, while working in the fields.

www.insightsactivelearn.com 73
Total Marks : 200
Test-24 (Subject)
( INSTA Prelims Test Series 2021 )

Xorai:

Made of bell-metal, the xorai — essentially a tray with a stand at the bottom, with or without a
cover — can be found in every Assamese household. While it is primarily used as an offering
tray during prayers, or to serve tamale-paan (betel-nut) to guests, a xorai is also presented
along with the jaapi and gamosa while felicitating someone. The bulk of xorais in Assam are
made in the state’s bell metal hub Sarthebari in Bajali district.

https://indianexpress.com/article/explained/explained-the-significance-of-jaapi-xorai-and-gamos
a-in-assam-poll-battle-7236796/

81. Consider the following statements regarding appointment of Retired Judge to Supreme Court
1. The chief justice of India can request a retired judge of the Supreme Court to act as a judge of
the Supreme Court for a temporary period.
2. After appointment, he will be deemed to be a judge of the Supreme Court.

Which of the statements given above is/are correct?


A. 1 only
B. 2 only
C. Both 1 and 2
D. Neither 1 nor 2

Correct Answer : A

Answer Justification :

Retired Judge

At any time, the chief justice of India can request a retired judge of the Supreme
Court or a retired judge of a high court (who is duly qualified for appointment as a
judge of the Supreme Court) to act as a judge of the Supreme Court for a temporary
period. He can do so only with the previous consent of the president and also of the person to
be so appointed. Such a judge is entitled to such allowances as the president may determine.
He will also enjoy all the jurisdiction, powers and privileges of a judge of Supreme Court. But,
he will not otherwise be deemed to be a judge of the Supreme Court.

Hence, statement 2 is incorrect.

82. Consider the following statements regarding Building and Other Construction Workers Act, 1996
1. Under the Act, the State Governments through their State Welfare Boards are mandated to
frame and implement welfare schemes for construction workers.
2. The fund, under the legislation, comprises the Cess @5% of construction costs which is levied
and collected by the Central Government.

www.insightsactivelearn.com 74
Total Marks : 200
Test-24 (Subject)
( INSTA Prelims Test Series 2021 )

Which of the statements given above is/are correct?


A. 1 only
B. 2 only
C. Both 1 and 2
D. Neither 1 nor 2

Correct Answer : A

Answer Justification :

The building and other construction workers (BOCW) are the most vulnerable segment of the
unorganized sector workers in India. They work under aggravating conditions with uncertain
future. A large chunk of them are migrant labourers working in different states far away from
their native places. They play a significant role in nation-building yet find themselves on the
margins of the society.

About the Building and Other Construction Workers Act, 1996:

Enacted to regulate the employment and conditions of service of these workers and to
provide for their safety, health and welfare measures.

Under the Act, the State Governments through their State Welfare Boards are
mandated to frame and implement welfare schemes for construction workers.

The fund, under the legislation, comprises the Cess @1% of construction costs
which is levied and collected by the State Governments and remitted to the
Welfare Fund. Hence, statement 2 is incorrect.

https://epaper.thehindu.com/Home/ShareArticle?OrgId=G7M8DPNN3.1&imageview=0

83. Suez Canal connects

A. Mediterranean Sea and Persian Gulf


B. Persian Gulf and the Red Sea
C. Mediterranean Sea and the Red Sea
D. Indian Ocean and the Red Sea

Correct Answer : C

Answer Justification :

www.insightsactivelearn.com 75
Total Marks : 200
Test-24 (Subject)
( INSTA Prelims Test Series 2021 )

Suez Canal:

Why in News?

A large cargo ship named ‘Ever Given’ got stuck near the southern end of the Suez Canal due
to a mishap caused by bad weather. This is causing a huge jam of vessels at either end of the
vital international trade artery.

Key facts:

The Suez Canal is an artificial sea-level waterway running north to south across the
Isthmus of Suez in Egypt, to connect the Mediterranean Sea and the Red Sea.

The canal separates the African continent from Asia.

It provides the shortest maritime route between Europe and the lands lying around the
Indian and western Pacific oceans.

It is one of the world’s most heavily used shipping lanes, carrying over 12% of world
trade by volume.

Hence, option (c) is correct.

https://www.hindustantimes.com/india-news/india-chalks-out-4-point-plan-to-deal-with-suez-ca
nal-blockage-101616805429223.html

84. Suez Canal Authority (SCA) is under the control of

www.insightsactivelearn.com 76
Total Marks : 200
Test-24 (Subject)
( INSTA Prelims Test Series 2021 )

A. Egypt
B. Israel
C. USA
D. United Kingdom

Correct Answer : A

Answer Justification :

The canal has been opened for navigation for the first time in Nov. 1869.

The canal is owned completely by the Egyptian government. No shares are available in the
stock market.

The canal is available for navigation all over the year.

The Suez Canal Authority (SCA), established on July 26th, 1956 , is a public and an
independent authority of a juristic personality, SCA shall report to the Prime
Minister. It has all the authorities needed for running the Canal without being limited by the
laws and the systems of the government.

Suez Canal Authority (SCA) is a state-owned authority which owns, operates and
maintains the Suez Canal. It was set up by the Egyptian government to replace the
Suez Canal Company in the 1950s which resulted in the Suez Crisis. Hence, option (a)
is correct.

The SCA manages, operates, uses, maintains and improves the Suez Canal. It is the SCA,
alone and exclusively, that issues and keeps in force the rules of navigation in the Canal and
other rules and regulations that provide for a well and orderly run canal.

The SCA may, when needed, establish, encourage or take part in establishing projects that are
related to the Canal. The SCA, for fulfilling its duties and obligations shall have all the relevant
authorities and in particular the authority to own, possess land and real estates.

The SCA shall impose and levy tolls on navigation and transit through the Canal, and on
pilotage, towage, berthing and other similar actions according to the laws and regulations.

The SCA has a separate budget that follows the rules applicable to the commercial projects.
The fiscal year starts on July 1st and ends on June 30th every year.

The SCA shall not take any procedure that go against the provisions of
the Constantinople Convention of 1888 concerning the free navigation of the Suez
Maritime Canal nor shall it give any privilege to a vessel or normal / legal person that
is not given, in the same circumstances, to other vessels or normal / legal persons and
nor shall it discriminate against some clients in favor of some other clients.​​

https://www.suezcanal.gov.eg/English/About/SuezCanalAuthority/Pages/SCAOverview.aspx

www.insightsactivelearn.com 77
Total Marks : 200
Test-24 (Subject)
( INSTA Prelims Test Series 2021 )

85. Consider the following statements regarding Contempt of court


1. The Attorney General’s consent is mandatory when supreme court wants to initiate a case of
contempt of court against a person.
2. The Contempt of Courts Act 1971 defines civil and criminal contempt and lays down the
powers and procedures by which courts can penalise contempt.

Which of the statements given above is/are correct?


A. 1 only
B. 2 only
C. Both 1 and 2
D. Neither 1 nor 2

Correct Answer : B

Answer Justification :

Attorney General of India K.K. Venugopal has denied consent to the initiation of contempt
proceedings against Congress MP Rahul Gandhi on the basis of a plea that he scandalised the
judiciary in an interview.

The Attorney General said no specific mentions were made about the Supreme Court or
its judges.

What is the law on contempt of courts?

The Contempt of Courts Act 1971 defines civil and criminal contempt, and lays down
the powers and procedures by which courts can penalise contempt, as well as the
penalties that can be given for the offence of contempt.

Contempt of court is the offense of being disobedient to or disrespectful toward


a court of law and its officers in the form of behavior that opposes or defies the
authority, justice and dignity of the court.

Why is the consent of the Attorney General required to initiate contempt


proceedings?

The objective behind requiring the consent of the Attorney General before taking
cognizance of a complaint is to save the time of the court.

This is necessary because judicial time is squandered if frivolous petitions are made
and the court is the first forum for bringing them in.

www.insightsactivelearn.com 78
Total Marks : 200
Test-24 (Subject)
( INSTA Prelims Test Series 2021 )

The AG’s consent is meant to be a safeguard against frivolous petitions, as it is


deemed that the AG, as an officer of the court, will independently ascertain whether
the complaint is indeed valid.

Under what circumstances is the AG’s consent not needed?

The AG’s consent is mandatory when a private citizen wants to initiate a case of
contempt of court against a person.

However, when the court itself initiates a contempt of court case the AG’s consent is
not required. Hence, statement 1 is incorrect.

Article 129 of the Constitution gives the Supreme Court the power to initiate contempt
cases on its own, independent of the motion brought before it by the AG or with the consent of
the AG.

https://epaper.thehindu.com/Home/ShareArticle?OrgId=GUJ8DMB11.1&imageview=0

86. Which of the following rights are protected under Consumer Protection Act, 2019?
1. Right to Safety.
2. Right to Consumer Education.
3. Right to be heard.
4. Right to Choose.

Select the correct answer using the code given below:


A. 1 and 4 only
B. 2 and 3 only
C. 2, 3 and 4 only
D. 1, 2, 3 and 4

Correct Answer : D

Answer Justification :

All the above statements are correct.

Consumer Protection Act, 2019

Context:

The Consumer Protection Act, 2019 has come into effect from July 20, replacing the
earlier Consumer Protection Act, 1986.

The Consumer Protection Bill, 2019 got the President’s nod on August 2019.

www.insightsactivelearn.com 79
Total Marks : 200
Test-24 (Subject)
( INSTA Prelims Test Series 2021 )

Highlights of the legislation:

1. Definition of consumer:

A consumer is defined as a person who buys any good or avails a service for a
consideration.

It does not include a person who obtains a good for resale or a good or service
for commercial purpose.

It covers transactions through all modes including offline, and online through
electronic means, teleshopping, multi-level marketing or direct selling.

1. Six consumer rights have been defined in the act, including the right to:

Right to Safety.

Right to be Informed.

Right to Choose.

Right to be heard.

Right to seek Redressal.

Right to Consumer Education.

1. Central Consumer Protection Authority:

The central government will set up CCPA to promote, protect and enforce the rights of
consumers.

It will regulate matters related to violation of consumer rights, unfair trade


practices, and misleading advertisements.

The CCPA will have an investigation wing, headed by a Director-General, which may
conduct inquiry or investigation into such violations.

www.insightsactivelearn.com 80
Total Marks : 200
Test-24 (Subject)
( INSTA Prelims Test Series 2021 )

https://www.insightsonindia.com/2020/07/22/consumer-protection-act-2019/

87. Pakal Dul Hydro Electric Project is proposed to be built on

A. Indus River
B. Chenab River
C. Sutlej River
D. Beas River

Correct Answer : B

Answer Justification :

Pakal Dul Hydro Electric Project:

The Pakal Dul Hydro Electric Project (1,000 MW) is proposed on the Marusudar river, a
tributary of the Chenab river, in Kishtwar district in Jammu and Kashmir. Hence, option
(b) is
correct.https://www.newindianexpress.com/nation/2021/mar/23/india-justifies-designs-of-paka
l-dul-lower-kalnai-hydro-projects-in-j-k-as-pakistan-objects-2280521.html

88. Indira Gandhi Memorial Tulip garden is located in

A. Jammu
B. Gangtok
C. Srinagar
D. Ladakh

Correct Answer : C

Answer Justification :

Tulip garden:

Also known as the Indira Gandhi Memorial Tulip garden, it is a tulip garden in Srinagar,
Jammu and Kashmir, India.

It is the largest tulip garden in Asia spread over an area of about 30 ha (74 acres).

The garden is located in the foothills of the Zaberwan range.

www.insightsactivelearn.com 81
Total Marks : 200
Test-24 (Subject)
( INSTA Prelims Test Series 2021 )

Hence, option (c) is correct.

https://indianexpress.com/photos/india-news/jammu-and-kashmir-tulip-garden-opens-for-tourist
s-7244322/4/

89. Consider the following statements regarding GST compensation


1. 101st Constitution Amendment Act, 2016 provides for provision to compensate the States for
loss of revenue arising out of implementation of the GST.
2. The GST Compensation Act, 2017 guaranteed States that they would be compensated for any
loss of revenue in the first five years of GST implementation.

Which of the statements given above is/are correct?


A. 1 only
B. 2 only
C. Both 1 and 2
D. Neither 1 nor 2

Correct Answer : C

Answer Justification :

All the above statements are correct.

What is the GST compensation?

The Constitution (One Hundred and First Amendment) Act, 2016, was the law which
created the mechanism for levying a nationwide GST. Written into this law was a
provision to compensate the States for loss of revenue arising out of implementation
of the GST. The adoption of the GST was made possible by the States ceding almost all their
powers to impose local-level indirect taxes and agreeing to let the prevailing multiplicity of
imposts be subsumed under the GST. While the States would receive the SGST (State GST)
component of the GST, and a share of the IGST (Integrated GST), it was agreed that revenue

www.insightsactivelearn.com 82
Total Marks : 200
Test-24 (Subject)
( INSTA Prelims Test Series 2021 )

shortfalls arising from the transition to the new indirect taxes regime would be made good
from a pooled GST Compensation Fund for a period of five years that is set to end in 2022.
This corpus in turn is funded through a compensation cess that is levied on so-called ‘demerit’
goods. The computation of the shortfall — the mechanism for which is spelt out in Section 7 of
the GST (Compensation to States) Act, 2017 — is done annually by projecting a revenue
assumption based on 14% compounded growth from the base year’s (2015-2016) revenue and
calculating the difference between that figure and the actual GST collections in that year. For
the 2020-21 fiscal year, the revenue shortfall has been anticipated at ₹3 lakh crore, with the
Compensation Fund expected to have only about ₹65,000 crore through cess accruals and
balance to pay the compensation to the States.

Why should the Centre pay states for GST loss?

The GST Compensation Act, 2017 guaranteed States that they would be compensated
for any loss of revenue in the first five years of GST implementation, until 2022, using
a cess levied on sin and luxury goods.

However, the economic slowdown has pushed both GST and cess collections down over
the last year, resulting in a 40% gap last year between the compensation paid and cess
collected.

States are likely to face a GST revenue gap of ₹3 lakh crore this year, as the economy
may contract due to COVID-19, which Finance Minister Nirmala Sitharaman termed an
unforeseen “act of God”.

What is compensation cess?

The modalities of the compensation cess were specified by the GST (Compensation to
States) Act, 2017.

This Act assumed that the GST revenue of each State would grow at 14% every year, from the
amount collected in 2015-16, through all taxes subsumed by the GST.

A State that had collected tax less than this amount in any year would be
compensated for the shortfall. The amount would be paid every two months based on
provisional accounts, and adjusted every year after the State’s accounts were audited by
the Comptroller and Auditor General.

This scheme is valid for five years, i.e., till June 2022.

Compensation cess fund:

A compensation cess fund was created from which States would be paid for any
shortfall. An additional cess would be imposed on certain items and this cess would be used
to pay compensation.

www.insightsactivelearn.com 83
Total Marks : 200
Test-24 (Subject)
( INSTA Prelims Test Series 2021 )

The items are pan masala, cigarettes and tobacco products, aerated water, caffeinated
beverages, coal and certain passenger motor vehicles.

The GST Act states that the cess collected and “such other amounts as may be
recommended by the [GST] Council” would be credited to the fund.

https://www.thehindu.com/business/Economy/what-is-the-gst-compensation-due-to-states/articl
e32531827.ece

90. Which of the following are the features of Places of Worship Act?
1. It says no person shall convert any place of worship of any religious denomination into one of a
different denomination or section.
2. Its objective is to freeze the status of any place of worship as it existed on August 15, 1947.

Which of the statements given above is/are correct?


A. 1 only
B. 2 only
C. Both 1 and 2
D. Neither 1 nor 2

Correct Answer : C

Answer Justification :

All the above statements are correct.

Places of Worship Act:

Context:

A co-mutawalli of 350-year-old ”TeelyWali masjid” of Lucknow, Wasif Hasan, has approached


the Supreme Court seeking intervention in the pending PIL challenging provisions of the
Places of Worship Act.

The applicant has opposed the plea claiming that the petition is a mischievous one which
aims at isolating the Muslim Community as a separate category from other religious
communities in India.

What’s the issue?

A petition filed in the court challenged a special law — Places of Worship (Special Provisions)
Act of 1991 — which freezes the status of places of worship as it was on August 15, 1947.

www.insightsactivelearn.com 84
Total Marks : 200
Test-24 (Subject)
( INSTA Prelims Test Series 2021 )

The petitioner had termed the law as “arbitrary, irrational and retrospective”.

What are the objectives of the Act?

To freeze the status of any place of worship as it existed on August 15, 1947.

To provide for the maintenance of the religious character of such a place of worship as
on that day.

To pre-empt new claims by any group about the past status of any place of worship and
attempts to reclaim the structures or the land on which they stood.

Key features:

The Act declares that the religious character of a place of worship shall continue to be
the same as it was on August 15, 1947.

It says no person shall convert any place of worship of any religious


denomination into one of a different denomination or section.

It declares that all suits, appeals or any other proceedings regarding converting the
character of a place of worship, which are pending before any court or authority on
August 15, 1947, will abate as soon as the law comes into force. No further legal
proceedings can be instituted.

https://epaper.thehindu.com/Home/ShareArticle?OrgId=GFG8DEP4F.1&imageview=0

91. On tap licensing provided RBI is

A. It means the RBI window for granting banking licences will be open only on specific
dates.
B. It means the RBI window for granting banking licences will be open throughout the
year.
C. It means the RBI window for granting banking licences will open only when a proposal
arises.
D. None of the above

www.insightsactivelearn.com 85
Total Marks : 200
Test-24 (Subject)
( INSTA Prelims Test Series 2021 )

Correct Answer : B

Answer Justification :

Panel to evaluate applications for Universal Banks, Small Finance Banks:

Context:

The Reserve Bank of India (RBI) has announced the creation of a Standing External Advisory
Committee under the chairmanship of Shyamala Gopinath for evaluating applications for
Universal Banks and Small Finance Banks.

This is part of the central bank’s earlier announced plan to give banking permits on a
continuous basis to candidates, a process that is is commonly known as ‘on-tap’
licensing.

What is On tap licensing?

It means the RBI window for granting banking licences will be open throughout the
year. Hence, option (b) is correct.

What is Universal banking?

It is a system of banking where banks undertake a blanket of financial services like


investment banking, commercial banking, development banking, insurance and other financial
services including functions of merchant banking, mutual funds, factoring, housing finance,
insurance etc.

https://www.thehindu.com/business/rbi-sets-up-external-committee-for-evaluating-applications-
for-universal-small-finance-banks/article34135398.ece

92. The Jury to select Gandhi Peace Prize comprises of


1. Prime Minister
2. Chief Justice of India
3. Leader of the single largest Opposition party in the Lok Sabha
4. Speaker
5. Founder of Sulabh International

Select the correct answer using the code given below:


A. 1, 2 and 4 only
B. 1, 2, 3 and 4 only
C. 2, 4 and 5 only
D. 1, 2, 3, 4 and 5

www.insightsactivelearn.com 86
Total Marks : 200
Test-24 (Subject)
( INSTA Prelims Test Series 2021 )

Correct Answer : D

Answer Justification :

All the above statements are correct.

Prime Minister Narendra Modi and comprising the Chief Justice of India, the leader of the
single largest Opposition party in the Lok Sabha, Lok Sabha Speaker and founder of Sulabh
International.

Gandhi Peace Prize:

The father of the nation of Bangladesh Sheikh Mujibur Rahman and the former Sultan of
Oman, the late Qaboos bin Said Al Said, will be awarded the Gandhi Peace Prize for
2020 and 2019, respectively.

They were selected by the jury, chaired by Prime Minister Narendra Modi and
comprising the Chief Justice of India, the leader of the single largest Opposition
party in the Lok Sabha, Lok Sabha Speaker and founder of Sulabh International.

About the Gandhi Peace Prize:

Instituted in the year 1995 on the occasion of the 125th birth anniversary of Mahatma
Gandhi.

This annual award is given to individuals and institutions for their contributions towards
social, economic and political transformation through non-violence and other Gandhian
methods.

The award carries a cash prize of Rs 1 crore, a citation and a Plaque as well as an
exquisite traditional handicraft/handloom item.

It is open to all persons regardless of nationality, creed, race or sex.

https://indianexpress.com/article/india/gandhi-peace-prize-2020-conferred-on-sheikh-mujibur-r
ahman-2019-prize-for-late-sultan-of-oman-7240361/

93. Who among the following are co organizers of World Summit on Information Society Forum 2021?
1. International Telecommunications Union

www.insightsactivelearn.com 87
Total Marks : 200
Test-24 (Subject)
( INSTA Prelims Test Series 2021 )

2. UNESCO
3. UNDP
4. UNCTAD

Select the correct answer using the code given below:


A. 1, 2 and 3 only
B. 2, 3 and 4 only
C. 1, 2 and 4 only
D. 1, 2, 3 and 4

Correct Answer : D

Answer Justification :

All the above statements are correct.

World Summit on Information Society Forum 2021:

It represents one of the world’s largest annual gatherings of the ‘ICT for development’
community.

It is co-organized by the International Telecommunications Union (ITU), UNESCO,


UNDP and UNCTAD.

The Forum has proven to be an efficient mechanism for coordination of multi-


stakeholder implementation activities, information exchange, creation of knowledge,
sharing of best practices.

World Summit on the Information Society Forum 2021

The World Summit on the Information Society Forum 2021 represents the world's largest
annual gathering of the ‘ICT for development’ community. The WSIS Forum, co-organized by
ITU, UNESCO, UNDP and UNCTAD, in close collaboration with all WSIS Action Line
Facilitators/Co-Facilitators, has proven to be an efficient mechanism for coordination of multi-
stakeholder implementation activities, information exchange, creation of knowledge, sharing
of best practices and continues to provide assistance in developing multi-stakeholder and
public/private partnerships to advance development goals. This Forum will provide structured
opportunities to network, learn and participate in multi-stakeholder discussions and
consultations on WSIS implementation. The Agenda and Programme of the Forum will be built
on the basis of the submissions received during the Open Consultation Process.

Furthermore, the 2021 WSIS Forum will provide an opportunity to serve as a platform to track
the achievements of WSIS Action Lines in collaboration with the UN Agencies involved and
provide information and analyses of the implementation of WSIS Action Lines since 2005.

www.insightsactivelearn.com 88
Total Marks : 200
Test-24 (Subject)
( INSTA Prelims Test Series 2021 )

https://www.itu.int/net4/wsis/forum/2021/en

94. Consider the following statements regarding Periodic Labour Force Survey (PLFS)
1. The survey for rural households is done annually.
2. The survey for urban households is done on a quarterly basis.
3. The survey was launched in 2017.

Which of the statements given above are correct?


A. 1 and 2 only
B. 2 and 3 only
C. 1 and 3 only
D. 1, 2 and 3

Correct Answer : D

Answer Justification :

All the above statements are correct.

Periodic Labour Force Survey (PLFS)

PLFS is India’s first computer-based survey which gives estimates of key employment
and unemployment indicators like the labour force participation rate, worker population
ratio, proportion unemployed and unemployment rate in rural households annually and on
a quarterly basis for the urban households.

The PLFS also gives the distribution of educated and unemployed people, which in turn
can be used as a basis for skilling of youth to make them more employable by industry.

The survey was launched in 2017 and the first annual report was released (July 2017-
June 2018), covering both rural and urban areas, in May 2019.

Definitions:

Labour Force Participation Rate: It is the percentage of people in the labour force (those
who are working or seeking or available for work) in the population.

Worker Population Ratio is the percentage of employed people.

Unemployment rate shows the percentage of people unemployed among the labour force.

Unemployed: A person who is unable to get work for even an hour in the last seven days
despite seeking employment is considered unemployed.

www.insightsactivelearn.com 89
Total Marks : 200
Test-24 (Subject)
( INSTA Prelims Test Series 2021 )

Key findings:

1. India’s unemployment rate fell between July 2018 and June 2019 to 5.8% from
6.1% during the same period of 2017-18, even as the labour force participation rate
rose to 37.5% from 36.9%.

2. The worker population ratio also increased, to 35.3% as against 34.7% in the
2017-18.

3. Urban unemployment rate reduced to 7.7% in 2018-19 from 7.8% and in rural India
to 5% from 5.3%.

4. Female participation rate improved in both urban and rural India during the
period under review, going up to 18.6% in 2018-19 from 17.5% the year before.

Challenges ahead:

To retain the falling trend in unemployment rate in 2019-20, which has witnessed a long
nationwide lockdown between March and June to prevent the spread of Covid-19, resulting in
job losses across sectors.

According to monthly data from the Centre for Monitoring Indian Economy, unemployment
rate in India shot up significantly from 7.87% in June 2019 to 23.48% in May 2020.

https://pib.gov.in/PressReleaseIframePage.aspx?PRID=1629366

95. Consider the following statements regarding surakshit matritva aashwasan (suman)
1. This initiative focuses on assured delivery of maternal and newborn healthcare services
encompassing wider access to free.
2. Under the scheme, the beneficiaries visiting public health facilities are entitled to several free
services.
3. All pregnant women, newborns and mothers up to 9 months of delivery will be able to avail
several free health care services.

Which of the statements given above are correct?


A. 1 and 2 only
B. 2 and 3 only
C. 1 and 3 only
D. 1, 2 and 3

Correct Answer : A

www.insightsactivelearn.com 90
Total Marks : 200
Test-24 (Subject)
( INSTA Prelims Test Series 2021 )

Answer Justification :

SURAKSHIT MATRITVA AASHWASAN (SUMAN)

An Initiative for Zero Preventable Maternal and Newborn Deaths.

This initiative focuses on assured delivery of maternal and newborn healthcare


services encompassing wider access to free, and quality services, zero tolerance for denial
of services, assured management of complications along with respect for women’s autonomy,
dignity, feelings, choices and preferences, etc.

About the scheme:

It aims to provide dignified and quality health care at no cost to every woman and newborn
visiting a public health facility.

Under the scheme, the beneficiaries visiting public health facilities are entitled to
several free services.

These include at least four ante natal check-ups that also includes one checkup during the
1st trimester, at least one checkup under Pradhan Mantri Surakshit Matritva Abhiyan, Iron
Folic Acid supplementation, Tetanus diptheria injection.

Eligibility:

All pregnant women, newborns and mothers up to 6 months of delivery will be able to
avail several free health care services. Hence, statement 3 is incorrect.

Features and benefits:

The scheme will enable zero expense access to the identification and management of
complications during and after the pregnancy.

The government will also provide free transport to pregnant women from home to the
health facility and drop back after discharge (minimum 48 hrs).

The pregnant women will be able to avail a zero expense delivery and C-section facility
in case of complications at public health facilities.

The scheme will ensure that there is zero-tolerance for denial of services to such
patients.

Need for and significance of the scheme:

The scheme aims to bring down the maternal and infant mortality rates in the nation and to

www.insightsactivelearn.com 91
Total Marks : 200
Test-24 (Subject)
( INSTA Prelims Test Series 2021 )

stop all preventable maternal and newborn deaths.

The scheme provides a positive and stress-free birth experience to the mother and newborn.

https://suman.nhp.gov.in/

96. Consider the following statements regarding National Mission on Libraries (NML)
1. The scheme was initiated by Ministry of Education.
2. It was formulated on the recommendation of National Knowledge Commission.
3. Raja Rammohun Roy Library Foundation (RRRLF) is the nodal agency for the implementation.

Which of the statements given above are correct?


A. 1 and 2 only
B. 2 and 3 only
C. 1 and 3 only
D. 1, 2 and 3

Correct Answer : B

Answer Justification :

National Mission on Libraries (NML)

The Union Ministry of Culture in 2012 formulated the NML scheme in


pursuance of National Knowledge Commission recommendations for sustained
attention for development of Libraries and Information Science Sector. Hence,
statement 1 is incorrect.

The National Knowledge Commission was constituted in 2005 to prepare a


blueprint for reform of knowledge related institutions and infrastructure which
would enable India to meet the challenges of the future.

The scheme consists of four components:

Creation of National Virtual Library of India (NVLI)

Setting up of NML Model Libraries

Quantitative & Qualitative Survey of Libraries

www.insightsactivelearn.com 92
Total Marks : 200
Test-24 (Subject)
( INSTA Prelims Test Series 2021 )

Capacity Building

The setting up on NML Model Libraries would develop 6 Libraries under Ministry of
Culture, 35 State Central Libraries and 35 District Libraries with particular emphasis on
economically backward districts, as model Libraries.

In addition, 629 district libraries across the states would be provided network
connectivity.

The Quantitative & Qualitative Survey of Libraries would be undertaken to prepare a


baseline data of libraries in India through a survey of 5000 Libraries.

The Capacity Building would be undertaken to enhance the professional competence of


library personnel.

Kolkata-based Raja Rammohun Roy Library Foundation (RRRLF) is the nodal agency
for the implementation of NML, a 10-member body headed by Prof. Deepak Pental, former
Vice Chancellor of Delhi University.

https://pib.gov.in/newsite/PrintRelease.aspx?relid=102951

97. World Development Report was released by

A. World Bank
B. International Monetary Fund
C. World Economic Forum
D. World Trade Organization

Correct Answer : A

Answer Justification :

World Development Report 2021: DATA FOR BETTER LIVES

Today’s unprecedented growth of data and their ubiquity in our lives are signs that the data
revolution is transforming the world. And yet much of the value of data remains untapped.
Data collected for one purpose have the potential to generate economic and social value in
applications far beyond those originally anticipated. But many barriers stand in the way,
ranging from misaligned incentives and incompatible data systems to a fundamental lack of
trust. World Development Report 2021: Data for Better Lives explores the tremendous
potential of the changing data landscape to improve the lives of poor people, while also

www.insightsactivelearn.com 93
Total Marks : 200
Test-24 (Subject)
( INSTA Prelims Test Series 2021 )

acknowledging its potential to open back doors that can harm individuals, businesses, and
societies. https://www.worldbank.org/en/publication/wdr2021

Hence, option (a) is correct.

98. Shigmo or Shigmotsav festival is organized in the state of

A. Odisha
B. Jharkhand
C. Chhattisgarh
D. Goa

Correct Answer : D

Answer Justification :

Shigmo festival is the spring festival in Goa celebrated by the Konkani diaspora of Goa, with a
lot of pompous and splendour. This festival takes place in the Phalguna month, according to
the Saka calendar of the Hindu community living in Goa. This festival is celebrated around
March every year.It is related to the Hindu lunar calendar and hence its date according to the
Gregorian calendar varies.

Hence, option (d) is correct.

What is Shigmo or Shigmotsav?

Shigmo is the celebration of a ‘rich, golden harvest of paddy’ by the tribal communities of Goa,
says Professor of History at Panaji’s Dhempe College of Arts and Science Prajal Sakhardande.
Agricultural communities including the Kunbis, Gawdas and Velips celebrate the festival that
also marks the onset of spring. Shigmo celebrations last over a fortnight in the months of
Phalgun-Chaitra months of the Hindu calendar that correspond with March-April every year.

What are the various activities that take place during Shigmo celebrations?

The festival begins with ‘Naman’ that is the invocation of the local folk deities on the village
‘maand’ or the village stage to the beats of percussion instruments like the Ghumat, Dhol,
Mhadle and Tashe by the male folk, says Sakhardande. This is called the ‘romta mell’ that
moves from one village to another. The celebration is replete with traditional, colourful
costumes, mythological installations, painted faces and costumes of various hues.

https://indianexpress.com/article/explained/explained-shigmo-festivities-goa-change-covid-19-7
243176/

99. As part of first inter-state tiger relocation project, the tigers were relocated to which of the following national park?

www.insightsactivelearn.com 94
Total Marks : 200
Test-24 (Subject)
( INSTA Prelims Test Series 2021 )

A. Satkosia Tiger Reserve


B. Kanha Tiger Reserve
C. Bandhavgarh National Park
D. Sundarbans

Correct Answer : A

Answer Justification :

Why the first inter-state tiger relocation project failed

Five-year-old tigress Sundari spent 28 months in captivity in Satkosia Tiger Reserve, Odisha
before she was tranquillised on Tuesday and relocated to Madhya Pradesh.

Sundari — a tigress shifted as part of India’s first inter-state translocation project in


2018 from Madhya Pradesh to Odisha — returned home. The five-year-old tigress
spent 28 months in captivity in Satkosia Tiger Reserve, Odisha as the two states
lingered on the process for her relocation despite the National Tiger Conservation
Authority (NTCA) shelving off the much-vaunted inter-state tiger translocation drive.
Sundari was tranquillised on Tuesday and caged before being relocated to MP. Hence,
option (a) is correct.

What was the Tiger Relocation Project?

The tiger relocation project was initiated in 2018 wherein two big cats, a male (Mahavir) from
Kanha Tiger Reserve and a female (Sundari) from Bandhavgarh from Madhya Pradesh were
relocated to Satkosia Tiger Reserve in Odisha, to shore up the tiger population in the state.
Sundari was brought to Satkosia a week after Mahavir’s arrival. The relocation was meant to
serve two purposes — reducing tiger population in areas with excess tigers to majorly reduce
territorial disputes, second, to reintroduce tigers in areas where the population has
considerably reduced due to various reasons.

What is the Satkosia Tiger Reserve and why was it chosen?

Encompassing an area of 963.87 sq km, the Satkosia Tiger Reserve spreads across four
districts and has as its core area 523 sq km. According to NTCA, Satkosia falls under reserves
where “there is a potential for increasing tiger populations”. Declared as a Tiger Reserve in
2007, Satkosia had a population of 12 tigers then. The numbers reduced to two in 2018. The
purpose of the relocation was to repopulate tigers in the reserve areas.

https://indianexpress.com/article/explained/explained-why-the-first-inter-state-tiger-relocation-
project-failed-7243200/

100. The IMF’s South Asia Training and Technical Assistance Center (SARTTAC) provides training to which of
the following countries?
1. Bhutan
www.insightsactivelearn.com 95
Total Marks : 200
Test-24 (Subject)
( INSTA Prelims Test Series 2021 )

2. Maldives
3. Sri Lanka
4. Pakistan

Select the correct answer using the code given below:


A. 1, 2 and 3 only
B. 2, 3 and 4 only
C. 1, 2 and 4 only
D. 1, 3 and 4 only

Correct Answer : A

Answer Justification :

The IMF’s South Asia Training and Technical Assistance Center (SARTTAC) was
officially inaugurated by Secretary Shaktikanta Das of India’s Ministry of Finance in
New Delhi on February 13, 2017. Senior officials from the Center’s six South Asian
member countries (Bangladesh, Bhutan, India, Maldives, Nepal, Sri Lanka) attended
along with development partners and an IMF delegation led by Deputy Managing
Director Carla Grasso. Hence, option (a) is correct.

SARTTAC is a collaborative venture between the IMF, the member countries, and development
partners. The Center’s strategic goal is to help its member countries strengthen their
institutional and human capacity to design and implement macroeconomic and financial
policies that promote growth and reduce poverty.

SARTTAC, the newest addition to the IMF’s global network of 14 regional centers, is a new
kind of capacity development institution, fully integrating formal training with hands-on
technical advice in a range of macroeconomic and financial areas, and generating synergies
between the two. SARTTAC is located in state-of-the-art facilities in New Delhi. The Center is
financed mainly by its six member countries, with additional support from the European
Union, the Republic of Korea, Australia and DIFD (UKaid).

https://www.sarttac.org/content/sarttac/en1/about-sarttac.html

www.insightsactivelearn.com 96

You might also like